SlideShare uma empresa Scribd logo
1 de 19
Baixar para ler offline
Earliest radiological sign of Pulmonary Venous Hypertension in Chest X-ray is:
A: Cephalization of pulmonary vascularity
B: Pleural effusion
C: Kerley B lines
D: Alveolar pulmonary edema
Correct Ans:A
Explanation
The first radiographic sign of pulmonary venous hypertension is cephalization of the
pulmonary vessels due to pulmonary vein and artery dilatation.
In the normal individual, there is continuous passage of fluid from the pulmonary veins into
adjacent interlobular lymphatics that return the fluid to the central mediastinal veins. If the
lympatic reserve is overcome by increased transudate as a result of elevated pulmonary
venous pressure, the interlobular septa are thickened and become visible radiographically.
The first radiologic sign of pulmonary venous hypertension is cephalization of the
pulmonary vessels.
Ref: Atlas of Pulmonary Vascular Imaging By Conard Wittram, Page 51; Textbook of
Radiology and Imaging By David Sutton, 5th Edition, Pages 288-89
Sample Previous Year Question on Radiological Signs based on previous Year
Questions of NEET PG, USMLE,PLAB,FMGE (MCI Screening). Please visit
www.medicoapps.org for more such Quizzes
A 8 year old boy is being treated for rickets. Which of the following investigations shows
the earliest evidence for healing?
A: Serum Ca
B: Serum phosphates
C: Radiological examination of long bones
D: Serum ALP
Correct Ans:C
Explanation
The earliest feature of healing in rickets is seen in X ray of growing end of the bones.
Sample Previous Year Question on Radiological Signs based on previous Year
Questions of NEET PG, USMLE,PLAB,FMGE (MCI Screening). Please visit
www.medicoapps.org for more such Quizzes
Allergic reactions to radiological contrast agents are:
A: Anaphylactic reactions
B:
IgE mediated
reactions
C: Urticaria
D: Edema
Correct Ans:A
Explanation
In radiological imaging it is usually believed that the adverse effects of contrast media are
non-IgE mediated anaphylactoid reactions, as a result of released vasoactive amines
(histamine, serotonin, etc) which trigger the mast cells directly. Since anaphylactoid
reaction is not given in the choice, anaphylactic reaction may be the right choice.
Ref: Encyclopedia of Diagnostic Imaging By Albert L Baert Pages 44-46
Sample Previous Year Question on Radiological Signs based on previous Year
Questions of NEET PG, USMLE,PLAB,FMGE (MCI Screening). Please visit
www.medicoapps.org for more such Quizzes
Which radiological procedure is used for studying vesico-ureteric reflux?
A: Ascending pyelogram
B:
Cystogra
m
C: Intravenous urogram
D: Micturition cystourethrogram
Correct Ans:D
Explanation
The diagnosis of vesico ureteric reflux is made using micturition cystourethrogram. Other
techniques used in diagnosing vesico ureteric reflux are simple or delayed cystography or
voiding cinefluoroscopy.
In a case of vesico ureteral reflux, cystogram may show one of the following findings:
Persistently dilated lower ureter, areas of dilatation in the ureter, ureter visualized
throughout its entire length shows, presence of hydroureteronephrosis with a narrow
juxtavesical ureteral segment or changes of healed pyelonephritis.
Ref: Smith's General Urology, 17e, Chapter 12
Sample Previous Year Question on Radiological Signs based on previous Year
Questions of NEET PG, USMLE,PLAB,FMGE (MCI Screening). Please visit
www.medicoapps.org for more such Quizzes
Radiological appearance of ‘thimble’ bladder is seen in the following condition:
A: Cystitis cystica
B: Chronic tuberculous cystitis
C: Neurogenic bladder
D: Acute tuberculous cystitis
Correct Ans:B
Explanation
Early tuberculosis of the bladder commences around the ureteric orifice or trigone, the
earliest evidence being pallor of the mucosa due to submucosal oedema. Subsequently,
tubercles may be seen and, in long standing cases, there is marked fibrosis and the capacity
of the bladder is greatly reduced giving the radiological appearance of ‘thimble bladder’.
Ref: Bailey & Love's Short Practice of Surgery 26E edited by
Norman Williams, Christopher Bulstrode, P Ronan O'Connell,
2013, Page 1327-1328
Sample Previous Year Question on Radiological Signs based on previous Year
Questions of NEET PG, USMLE,PLAB,FMGE (MCI Screening). Please visit
www.medicoapps.org for more such Quizzes
Routine radiological examination of a middle aged man shows "Spongy appearance" with
central sunburst calcification. This is seen in:
A: Pancreatic adenocarcinoma
B: Mucinous cystadenocarcinoma
C: Somatostatinoma
D: Serous cystadenoma
Correct Ans:D
Explanation
Serous cystadenoma is a rare solitary, benign cystic neoplasm of the pancreas. It consists of
multiple small locules lined by cuboidal epithelium, the cells of which contain abundant glycogen (also
called microcystic, serous, and glycogen-rich cystadenoma).
Radiological appearance of serous cystadenoma of pancreas shows characteristic central sunburst
calcificationwhich is pathognomonic.
Ref: The Pancreas: An Integrated Textbook of Basic Science, Medicine, and Surgery
edited by Hans-Gunther Beger, Markus Buchler, Richard Kozarek, Markus Lerch, John
Neoptolemos, Andrew Warshaw, David Whitcomb, Keiko Shiratori, 2009, Page 933.
Sample Previous Year Question on Radiological Signs based on previous Year
Questions of NEET PG, USMLE,PLAB,FMGE (MCI Screening). Please visit
www.medicoapps.org for more such Quizzes
Which of the following is the radiological finding of a benign gastric ulcer?
A: Hampton line
B: Collar at the neck of the ulcer
C: Projection from the lumen of the stomach
D: All of the above
Correct Ans:D
Explanation
The radiographic features that suggest a benign gastric ulcer include,
• Projection from the lumen of the stomach
• Smooth lucent line (Hampton line) or collar at the neck of the ulcer
• Normal rugal folds that radiate to the edge of the ulcer collection
• Complete and permanent healing of the ulcer on repeat radiographic or
endoscopic examination of the stomach
If at least two or more of these findings are present, a
confident radiographic diagnosis of benign gastric ulcer is
possible.
Ref: Ott D.J. (2011). Chapter 10. Gastrointestinal Tract. In M.Y. Chen, T.L. Pope, D.J. Ott
(Eds), Basic Radiology, 2e.
Sample Previous Year Question on Radiological Signs based on previous Year
Questions of NEET PG, USMLE,PLAB,FMGE (MCI Screening). Please visit
www.medicoapps.org for more such Quizzes
Which of the following statements best describes 'Background Radiation'?
A: Radiation in the background of nuclear reactors
B: Radiation in the background during radiological investigations
C: Radiation present constantly from natural sources
D: Radiation from nuclear fall out
Correct Ans:C
Explanation
Background radiation refers to those coming from the environment of natural radioactivity
at the earths surface and the those from direct cosmic radiation that arrives at the earths
surface.
Ref: Radiation Biophysics By Edward L. Alpen, Page 432; The Biological Basis of Nursing :
Cancer By William T. Blows, Page 54, 56
Sample Previous Year Question on Radiological Signs based on previous Year
Questions of NEET PG, USMLE,PLAB,FMGE (MCI Screening). Please visit
www.medicoapps.org for more such Quizzes
A 43 year old male during the preoperative evaluation of hernia repair is found to have a
solitary pulmonary nodule. What is the important radiological feature to suggest its benign
nature?
A: Large lesion with calcification
B: Irregular shape
C: No calcification
D: 1 cm lesion with uniform calcification
Correct Ans:D
Explanation
The important radiological characteristics to suggest benign lesion in solitary pulmonary
nodule:
• No growth in follow up imaging in two years and calcification
• Younger age group
• Regular shape
• Normal edge
Ref: Lange radiology, Edition - 7, Page - 256.
Sample Previous Year Question on Radiological Signs based on previous Year
Questions of NEET PG, USMLE,PLAB,FMGE (MCI Screening). Please visit
www.medicoapps.org for more such Quizzes
Which of the following radiological sign will be present in a young lady with symptoms
suggestive of pulmonary embolism?
A: Hampton's hump
B:
Westermark
sign
C: Fleischner sign
D: All of the above
Correct Ans:D
Explanation
Pulmonary infarction may occur if the pulmonary venous pressure is elevated or the
bronchial arterial supply to a region is deficient. The cone-shaped area of pulmonary
infarction has been called a Hampton's hump. An area of radiolucency, corresponding to
diminished pulmonary vascularity distal to a pulmonary embolism, is occasionally seen and
is called the Westermark sign. There may also be an increase in the size of the pulmonary
artery proximal to a large central pulmonary embolus (Fleischner sign).
Ref: Chiles C., Gulla S.M. (2011). Chapter 4. Radiology of the Chest. In M.Y. Chen, T.L. Pope,
D.J. Ott (Eds), Basic Radiology, 2e.
Sample Previous Year Question on Radiological Signs based on previous Year
Questions of NEET PG, USMLE,PLAB,FMGE (MCI Screening). Please visit
www.medicoapps.org for more such Quizzes
All of the following are radiological signs of scurvy, EXCEPT:
A: Trimmerfeld zone
B:
Frenkel's
line
C: Pelican spur
D: Soap bubble appearance
Correct Ans:D
Explanation
Ascorbic acid is essential for the hydroxylation of lysine and proline in collagen, hence the
impairment in wound healing. Collagen is an important part if bone also, so the defect in
bones occurs.
Soap bubble appearence in the xrays is usually seen in osteoblastoma and in gaint cell
tumour.
Ref: Terry Yochum, Lindsay Rowe (2005), Chapter 14, “Nutritional, Metabolic and Endocrine
disorders”, In the Book, “Essentials of Skeletal Radiology”, Volume 1, 3rd Edition, USA,
Page 1517 ; Text Book of Radiology and Imaging By Sutton, 7th Edition, Page 1356
Sample Previous Year Question on Radiological Signs based on previous Year
Questions of NEET PG, USMLE,PLAB,FMGE (MCI Screening). Please visit
www.medicoapps.org for more such Quizzes
Which of the following condition gives the characteristic radiological finding "bone within a
bone" appearance?
A: Osteogenesis imperfecta
B:
Osteopetrosi
s
C: Scurvy
D: Rickets
Correct Ans:B
Explanation
Radiologic examination of osteopetrosis shows increased bone density and transverse
bands in the shafts, clubbing of ends, and vertical striations of long bones. Thickening
about the cranial foramina is present, and heterotopic calcification of soft tissues is
possible. Other characteristic findings include a miniature bone inset within each vertebral
body (bone-within-a bone appearance) and increased density at the end plates (sandwich
vertebrae).
Ref: Clinical Imaging: An Atlas of Differential Diagnosis By Ronald L. Eisenberg, 2012, Page
1088.
Sample Previous Year Question on Radiological Signs based on previous Year
Questions of NEET PG, USMLE,PLAB,FMGE (MCI Screening). Please visit
www.medicoapps.org for more such Quizzes
20 years old female with complaints of nausea, vomiting, dizziness and pain in the legs. Her
physical examination and all lab investigations and radiological investigations are normal.
What would be the most probable diagnosis?
A: Generalized anxiety disorder
B:
Conversion
disorder
C: Somatoform pain disorder
D: Somatisation disorder
Correct Ans:D
Explanation
Here the patient presents with multiple, medically unexplained symptoms (MUS). This is an
example of Somatisation Disorder. In somatoform pain disorder the major symptom will be
unexplained chronic pain, which is usually isolated. Patients have a long history of
complaints of severe pain out of proportion to any biomedical findings that are present.
Exacerbations and remission of complaints correlate with psychogenic factors
The term somatization refers to the experience and reporting of physical symptoms that
cause distress but lack a corresponding level of tissue damage or pathology and are linked
to psychosocial stress.
In contrast to this broad and inclusive view of the process, psychiatrists have developed
strict diagnostic criteria that define several distinct disorders, which are collectively
referred to as the somatoform disorders. As such, clinicians should be careful to distinguish
between somatization, as defined above, and somatization disorder, which is one type of
somatoform disorder. In general, these conditions are chronic and reflect an enduring way
for the affected individuals to cope with psychosocial stressors.
Ref: Shim J., Eisendrath S.J. (2008). Chapter 25. Somatization. In M.D. Feldman, J.F.
Christensen (Eds), Behavioral Medicine: A Guide for Clinical Practice, 3e.
Sample Previous Year Question on Radiological Signs based on previous Year
Questions of NEET PG, USMLE,PLAB,FMGE (MCI Screening). Please visit
www.medicoapps.org for more such Quizzes
Radiological examination shows evidence of bone infarct in a child. She may have the
following condition:
A: Iron deficiency anaemia
B:
Thalassemi
a
C: Sickle cell disease
D: Hereditary spherocytosis
Correct Ans:C
Explanation
Sickle cell disease is frequently associated with bone infarction resulting from
vasoocclusion secondary to the sickling of red cells. Bone infarction also occurs in
hemoglobin sickle cell disease and sickle cell thalassemia. The bone pain in sickle cell crisis
is due to bone and bone marrow infarction.
In children, infarction of the epiphyseal growth plate interferes with normal growth of the
affected extremity. Radiographically, infarction of the bone cortex results in periosteal
elevation and irregular thickening of the bone cortex. Infarction in the bone marrow leads
to lysis, fibrosis, and new bone formation.
Ref: Langford C.A. (2012). Chapter 336. Arthritis Associated with Systemic Disease, and
Other Arthritides. In D.L. Longo, A.S. Fauci, D.L. Kasper, S.L. Hauser, J.L. Jameson, J.
Loscalzo (Eds), Harrison's Principles of Internal Medicine, 18e.
Sample Previous Year Question on Radiological Signs based on previous Year
Questions of NEET PG, USMLE,PLAB,FMGE (MCI Screening). Please visit
www.medicoapps.org for more such Quizzes
A 65-year-old man develops oliguria and peripheral edema over a period of weeks.
Urinalysis reveals hematuria and proteinuria; examination of the urinary sediment reveals
red cell casts. Radiological and ultrasound studies fail to demonstrate an obstructive lesion.
Renal biopsy shows many glomerular crescents. This presentation is most suggestive of
which of the following conditions?
A: Anti-glomerular basement membrane disease
B: Diabetic nephropathy
C: Hypertensive nephropathy
D: Lupus nephritis
Correct Ans:A
Explanation
The two principal causes of rapidly progressive glomerulonephritis are anti-glomerular
basement membrane (including both Goodpasture's syndrome and isolated anti-glomerular
basement disease) and primary systemic vasculitis (including Wegener's granulomatosis,
microscopic polyarteritis, idiopathic rapidly progressive glomerulonephritis, Churg-Strauss
syndrome, polyarteritis nodosa, giant-cell arteritis, and Takayasu's arteritis). A very large
variety of other systemic and primary glomerular disease may occasionally cause rapidly
progressive glomerulonephritis, but this is usually not the typical presentation for these
diseases.
Diabetic nephropathy typically begins with microalbuminuria and hypertension and
progresses over a 10 to 20 year period to renal failure.
Hypertensive nephropathy due to essential hypertension typically presents with slowly
rising BUN and creatinine; hypertensive nephropathy due to malignant hypertension
presents with more rapidly rising BUN and creatinine.
Lupus nephritis can have many presentations, but the most typical is proteinuria, which
may be severe enough to cause nephrotic syndrome. Also, 90% of cases of systemic lupus
erythematosus occur in women, usually of childbearing age.
Ref: Lewis J.B., Neilson E.G. (2012). Chapter 283. Glomerular Diseases. In D.L. Longo, A.S.
Fauci, D.L. Kasper, S.L. Hauser, J.L. Jameson, J. Loscalzo (Eds), Harrison's Principles of
Internal Medicine, 18e
Sample Previous Year Question on Radiological Signs based on previous Year
Questions of NEET PG, USMLE,PLAB,FMGE (MCI Screening). Please visit
www.medicoapps.org for more such Quizzes
Which of the following condition is associated with radiological ‘Spalding sign'?
A: Mummification
B:
Maceratio
n
C: Hanging
D: Drowning
Correct Ans:B
Explanation
‘Spalding sign' is a sign of maceration. Loss of alignment and overriding of the bones of the
cranial vault occur due to shrinkage of the cerebrum can be seen in X-ray after death of the
foetus. The sign will develop earlier with a vertex presentation than with a breech. It may
be detected within a few days of death of the foetus.
Signs of dead birth other than maceration are rigor mortis at delivery and mummification.
Ref: The Essentials of Forensic Medicine and Toxicology by KS Narayan Reddy, 27th edition,
Page 382-383.
Sample Previous Year Question on Radiological Signs based on previous Year
Questions of NEET PG, USMLE,PLAB,FMGE (MCI Screening). Please visit
www.medicoapps.org for more such Quizzes
What is the ICRP (International Commission on Radiological Protection)
recommended genetic dose of radiation exposure for general population?
A: 5 rems over a period of 30 years
B: 30 rems over a period of 30 years
C: 5 rems over a period of 5 years
D: 30 rems over a period of 5 years
Correct Ans:A
Explanation
ICRP recommends that genetic dose to population should not exceed 5 rems over
a period of 30 years.
Ref: Park’s textbook of Preventive and Social Medicine, 21st edition, page-687
Sample Previous Year Question on Radiological Signs based on previous Year
Questions of NEET PG, USMLE,PLAB,FMGE (MCI Screening). Please visit
www.medicoapps.org for more such Quizzes
One of the following disease will show urinary bladder calcification radiologically which
resembles fetal head in pelvis:
A: Tuberculosis
B: Schistosomiasis
C: Chronic cystitis
D: Malignancy
Correct Ans:B
Explanation
A plain film of the abdomen may show areas of grayness in the flank (enlarged
hydronephrotic kidney) or in the bladder area (large tumor). Opacifications (stones) may
be noted in the kidney, ureter, or bladder. Linear calcification may be seen in the ureteral
and bladder walls. Punctate calcification of the ureter (ureteritis calcinosa) and a
honeycombed calcification of the seminal vesicle may be obvious. The classic presentation
of a calcified bladder, which looks like a fetal head in the pelvis, is pathognomonic of
chronic urinary schistosomiasis.
Ref: Tanagho E.A., Kane C.J. (2008). Chapter 14. Specific Infections of the Genitourinary
Tract. In E.A. Tanagho, J.W. McAninch (Eds), Smith's General Urology, 17e.
Sample Previous Year Question on Radiological Signs based on previous Year
Questions of NEET PG, USMLE,PLAB,FMGE (MCI Screening). Please visit
www.medicoapps.org for more such Quizzes
All of the following are correct statements about radiological evaluation of a patient with
Cushing's syndrome, EXCEPT:
A: MRI of the sella tursica will always identify a pituitary cause for Cushing's syndrome
B:
Petrosal sinus sampling is the best way to distinguish a pituitary
tumor from an ectopic ACTH producing tumor
C: MRI of the adrenals may distinguish adrenal adenoma from carcinoma
D:
Adrenal CT scan distinguished adrenal cortical hyperplasia from
an adrenal tumor
Correct Ans:A
Explanation
Most ACTH-secreting pituitary tumors are <5 mm in diameter, and about half are undetectable by
sensitive MRI. So MRI is not a useful investigation in identifying pituitary cause of Cushing's
syndrome.
Ref: Harrison’s Internal Medicine, 18th Edition, Chapter 339; CURRENT Diagnosis and Treatment:
Surgery, 13th Edition, Chapter 33
Sample Previous Year Question on Radiological Signs based on previous Year
Questions of NEET PG, USMLE,PLAB,FMGE (MCI Screening). Please visit
www.medicoapps.org for more such Quizzes
Radiological appearance of ‘corkscrew oesophagus’ is a characteristic finding seen in the
condition:
A: Achalasia cardia
B: Diffuse esophageal spasm
C: Carcinoma esophagus
D: Reflux oesophagitis
Correct Ans:B
Explanation
The lower esophageal sphincter in patients with diffuse esophageal spasm (DES) usually
shows a normal resting pressure and relaxation on swallowing. A hypertensive sphincter
with poor relaxation may also be present. In patients with advanced disease, the
radiographic appearance of tertiary contractions appears helical, and has been
termed corkscrew esophagus or pseudodiverticulosis. Patients with segmental or diffuse
esophageal spasm can compartmentalize the esophagus and develop an epiphrenic or mid
esophageal diverticulum between two areas of high pressure occurring simultaneously.
Ref: Jobe B.A., Hunter J.G., Peters J.H. (2010). Chapter 25. Esophagus and Diaphragmatic
Hernia. In F.C. Brunicardi, D.K. Andersen, T.R. Billiar, D.L. Dunn, J.G. Hunter, J.B. Matthews,
R.E. Pollock (Eds), Schwartz's Principles of Surgery, 9e.
Sample Previous Year Question on Radiological Signs based on previous Year
Questions of NEET PG, USMLE,PLAB,FMGE (MCI Screening). Please visit
www.medicoapps.org for more such Quizzes
A 50 year old female is admitted with abdominal pain and anuria. Radiological studies
revealed bilateral impacted ureteric stones with hydronephrosis. Urine analysis showed
RBCs with pus cells in urine. Serum creatinine level was 16 mg/dl and urea level was 200
mmol/1. Which of the following should be the immediate treatment?
A: Hemodialysis
B: 'J' stent drainage
C: Lithotripsy
D: Ureteroscopic removal of stones
Correct Ans:B
Explanation
This is the best modality of treatment to immediately revert anuria in patients with bilateral
obstructing renal stone. Stenting can be done in other scenarios such as pyelonephritis due
to obstructed stone, severe renal colic, long term obstruction.
Ref: Urinary Stone Disease : The Practical Guide to Medical and Surgical Management By
Marshall L. Stoller, Maxwell V. Meng, 2007, Page 475
Sample Previous Year Question on Radiological Signs based on previous Year
Questions of NEET PG, USMLE,PLAB,FMGE (MCI Screening). Please visit
www.medicoapps.org for more such Quizzes
A 50 year old female is admitted with abdominal pain and anuria. Radiological studies
revealed bilateral impacted ureteric stones with hydronephrosis. Urine analysis showed
RBCs with pus cells in urine. Serum creatinine level was 16 mg/dl and urea level was 200
mmol/1. Which of the following should be the immediate treatment?
A: Hemodialysis
B: 'J' stent drainage
C: Lithotripsy
D: Ureteroscopic removal of stones
Correct Ans:B
Explanation
'J' stent drainage is the best modality of treatment to immediately revert anuria in patients
with bilateral obstruction of renal stone. Stenting can be done in other scenarios such as
pyelonephritis due to obstructed stone, severe renal colic, long term obstruction.
Sample Previous Year Question on Radiological Signs based on previous Year
Questions of NEET PG, USMLE,PLAB,FMGE (MCI Screening). Please visit
www.medicoapps.org for more such Quizzes
All of the following are correct statements about radiological evaluation of a patient with
Cushing's syndrome, EXCEPT:
A: MRI of the sella tursica will always identify a pituitary cause for Cushing's syndrome
B:
Petrosal sinus sampling is the best way to distinguish a pituitary
tumor from an ectopic ACTH producing tumor
C: MRI of the adrenals may distinguish adrenal adenoma from carcinoma
D:
Adrenal CT scan distinguished adrenal cortical hyperplasia from
an adrenal tumor
Correct Ans:A
Explanation
Most ACTH-secreting pituitary tumors are <5 mm in diameter, and about half are undetectable by
sensitive MRI. So MRI is not a useful investigation in identifying pituitary cause of Cushing's
syndrome.
Ref: Harrison’s Internal Medicine, 18th Edition, Chapter 339; CURRENT Diagnosis and Treatment:
Surgery, 13th Edition, Chapter 33
Sample Previous Year Question on Radiological Signs based on previous Year
Questions of NEET PG, USMLE,PLAB,FMGE (MCI Screening). Please visit
www.medicoapps.org for more such Quizzes
Which of the following is the best radiological investigation in a case of suspected splenic
rupture?
A: CT
B: USG
C: MRI
D: Peritoneal rupture
Correct Ans:A
Explanation
CT can clearly show the extend of splenic injury. It can also distinguish the viable portion of the
spleen.
Sample Previous Year Question on Radiological Signs based on previous Year
Questions of NEET PG, USMLE,PLAB,FMGE (MCI Screening). Please visit
www.medicoapps.org for more such Quizzes
A 45 year old female presented with a breast lump. You are
suspecting carcinoma in this patient and want to do a triple
assessment.
Assessment: Triple assessment is a combination of clinical assessment, radiological imaging and a
tissue sample taken for either cytological or histological analysis.
Reason: The positive predictive value of this combination will exceed 99.9%.
A: Both Assertion and Reason are true, and Reason is the correct explanation for Assertion
B:
Both Assertion and Reason are true, and Reason is not the
correct explanation for Assertion
C: Assertion is true, but Reason is false
D: Assertion is false, but Reason is true
Correct Ans:A
Explanation
In any patient who presents with a breast lump or other symptoms suspicious of
carcinoma, the diagnosis should be made by triple assessment.
Ref: Bailey & Love’s Short Practice of Surgery, 24th Edition, Page 826.
Sample Previous Year Question on Radiological Signs based on previous Year
Questions of NEET PG, USMLE,PLAB,FMGE (MCI Screening). Please visit
www.medicoapps.org for more such Quizzes
A 2 month old infant is presented with failure to thrive, recurrent emesis,
hepatosplenomegaly, and adrenal insufficiency. Adrenal calcification is noted radiologically.
What is the most likely diagnosis?
A: Adrenal hemorrhage
B:
Wolman's
disease
C: Pheochromocytoma
D: Addison's disease
Correct Ans:B
Explanation
This child is showing features of Wolman disease.
Wolman disease appear in the first few weeks of life and presents with persistent vomiting
and diarrhea, hepatosplenomegaly, xanthomatosis and adrenal calcification. Patients have
a complete absence of enzyme A of lysosome acid lipase.
Investigations: shows liver enzyme abnormalities, decreased adrenal responsiveness to
ACTH stimulation, normal or decreased plasma lipids. Cholesterol ester and triglyceride
deposition occur in the lysosomes of liver parenchymal and Kuppfer cells, and in
macrophages of adrenal gland, lymph node, intestinal mucosa etc.
Sample Previous Year Question on Radiological Signs based on previous Year
Questions of NEET PG, USMLE,PLAB,FMGE (MCI Screening). Please visit
www.medicoapps.org for more such Quizzes
Shenton’s line is a radiological line used to determine the pathology of:
A: Hip
B: Ankle
C: Elbow
D: Shoulder
Correct Ans:A
Explanation
Shenton’s line is an imaginary semi-circular line joining medial cortex of the femoral neck
to lower border of the superior pubic ramus. Shenton’s line is broken in posterior
dislocation of the hip.
Ref: Essential Orthopedics by Maheshwari, 3rd edition, Page 111.
Sample Previous Year Question on Radiological Signs based on previous Year
Questions of NEET PG, USMLE,PLAB,FMGE (MCI Screening). Please visit
www.medicoapps.org for more such Quizzes
A sixty five year old lady presents with a long standing history of pain and swelling in her
right knee. Pain is significantly interfering with her activities of daily living. Radiological
evaluation shows grade III changes of osteoarthritis. Which of the following is the
recommended line of management?
A: Conservative management
B: Arthroscopic lavage/washout
C: Partial knee replacement
D: Total knee replacement
Correct Ans:D
Explanation
This patients history strongly supports for a total knee replacement. As she is sixty five years old and
radiological reports showing evidence of grade III changes of osteoarthritis in this patient, the best
recommendation would be a Total knee arthroplasty. Patients with severe symptomatic Osteoarthritis
have failed to respond to medical therapy and have progressive limitations in their daily activities
should be referred for surgical options such as arthroplasty or joint reconstruction like osteotomy or
arthrodesis.
Ref: Osteoarthritis: Diagnosis and medical/surgical management, by Roland W.Moskowitz,
Page 402, 403.
Sample Previous Year Question on Radiological Signs based on previous Year
Questions of NEET PG, USMLE,PLAB,FMGE (MCI Screening). Please visit
www.medicoapps.org for more such Quizzes
A child with a history of pain over lower extremities since few moths shows radiological
evidence of osteosarcoma. Most common site of osteosarcoma is:
A:
Upper end of
femur
B: Lower end of femur
C: Lower end of fibula
D: Lower end of tibia
Correct Ans:B
Explanation
The classic form of osteosarcoma is typically seen in patients in their second or third
decade, with a peak in the adolescent growth spurt. It occurs more frequently in males than
in females and is found in the metaphyseal areas of long bones, with 50% of lesions seen
about the knee joint. The distal femur is the most common site, followed by the proximal
tibia and then the proximal humerus.
Ref: Randall R.L., Hoang B.H. (2006). Chapter 6. Musculoskeletal Oncology. In H.B. Skinner
(Ed), CURRENT Diagnosis & Treatment in Orthopedics, 4e.
Sample Previous Year Question on Radiological Signs based on previous Year
Questions of NEET PG, USMLE,PLAB,FMGE (MCI Screening). Please visit
www.medicoapps.org for more such Quizzes
Radiological features of left ventricular heart failure are all, EXCEPT:
A: Kerley B lines
B:
Cardiomegal
y
C: Oligemic lung fields
D: Increased flow in upper lobe veins
Correct Ans:C
Explanation
Classic signs of acute heart failure that can be seen on a chest x-ray of left ventricular
failure are:
• Cardiomegaly
• Upper lobe blood diversion
• "Bat's wing" alveolar edema
• Pleural effusions
Echocardiography is the investigation of choice and can identify and quantify LVH and
dysfunction (both systolic and diastolic) as well as examine causes of heart failure, such as
valve abnormalities.
Oligemic lung fields seen in Pulmonary atresia, stenosis; Ebstein’s anomaly.
Also Know:
Chest X ray Findings
• Boot shaped heart - TOF
• “3”like appearance - Coarctation of aorta.
• Figure of ‘8’- TAPVC(Total anomalous pulmonary venous connection)
• Bat wings appearance -Pulmonary edema.
Ref: O'Rourke R.A., Gilkeson R.C. (2011). Chapter 17. Cardiac Radiography. In V. Fuster,
R.A. Walsh, R.A. Harrington (Eds), Hurst's The Heart, 13e.
Sample Previous Year Question on Radiological Signs based on previous Year
Questions of NEET PG, USMLE,PLAB,FMGE (MCI Screening). Please visit
www.medicoapps.org for more such Quizzes

Mais conteúdo relacionado

Mais procurados

Ultrasound of Vascular anomalies by Oscar M. Navarro
Ultrasound of Vascular anomalies by Oscar M. NavarroUltrasound of Vascular anomalies by Oscar M. Navarro
Ultrasound of Vascular anomalies by Oscar M. NavarroDr. Naveed Quetta
 
Radiological approach to gastric ulcer disease
Radiological approach to gastric ulcer diseaseRadiological approach to gastric ulcer disease
Radiological approach to gastric ulcer diseaseNavneet Ranjan
 
Radiological imaging of pleural diseases
Radiological imaging of pleural diseases Radiological imaging of pleural diseases
Radiological imaging of pleural diseases Pankaj Kaira
 
Radiological imaging of mediastinal masses
Radiological imaging of mediastinal massesRadiological imaging of mediastinal masses
Radiological imaging of mediastinal massesPankaj Kaira
 
Acute Abdomen-Radiology
Acute Abdomen-RadiologyAcute Abdomen-Radiology
Acute Abdomen-RadiologyParvathy Nair
 
Radiology Spots PPT- 3 by Dr Chandni Wadhwani
 Radiology Spots PPT- 3 by Dr Chandni Wadhwani Radiology Spots PPT- 3 by Dr Chandni Wadhwani
Radiology Spots PPT- 3 by Dr Chandni WadhwaniChandni Wadhwani
 
Signs in Chest Xray
Signs in Chest Xray Signs in Chest Xray
Signs in Chest Xray Archana Koshy
 
Solitary pulmonary nodule
Solitary pulmonary noduleSolitary pulmonary nodule
Solitary pulmonary noduleNavni Garg
 
Imaging in Skull base
Imaging in Skull baseImaging in Skull base
Imaging in Skull baseRakesh Ca
 
Imaging of pulmonary embolism
Imaging of pulmonary embolismImaging of pulmonary embolism
Imaging of pulmonary embolismThorsang Chayovan
 
Ultrasound of spinal cord in neonates Dr. Muhammad Bin Zulfiqar
Ultrasound of spinal cord in neonates Dr. Muhammad Bin ZulfiqarUltrasound of spinal cord in neonates Dr. Muhammad Bin Zulfiqar
Ultrasound of spinal cord in neonates Dr. Muhammad Bin ZulfiqarDr. Muhammad Bin Zulfiqar
 
Presentation1.pptx, radiological signs in thoracic radiology.
Presentation1.pptx, radiological signs in thoracic radiology.Presentation1.pptx, radiological signs in thoracic radiology.
Presentation1.pptx, radiological signs in thoracic radiology.Abdellah Nazeer
 
INFLAMMATORY BOWEL DISEASE IMAGING(RADIOLOGY)
INFLAMMATORY BOWEL DISEASE IMAGING(RADIOLOGY)INFLAMMATORY BOWEL DISEASE IMAGING(RADIOLOGY)
INFLAMMATORY BOWEL DISEASE IMAGING(RADIOLOGY)Khyati Vadera
 
Information regarding e lora for AERB registration
Information regarding e lora for AERB registrationInformation regarding e lora for AERB registration
Information regarding e lora for AERB registrationDr Jitu Lal Meena
 

Mais procurados (20)

Ultrasound of Vascular anomalies by Oscar M. Navarro
Ultrasound of Vascular anomalies by Oscar M. NavarroUltrasound of Vascular anomalies by Oscar M. Navarro
Ultrasound of Vascular anomalies by Oscar M. Navarro
 
Lung cancer radiology
Lung cancer radiologyLung cancer radiology
Lung cancer radiology
 
Radiological approach to gastric ulcer disease
Radiological approach to gastric ulcer diseaseRadiological approach to gastric ulcer disease
Radiological approach to gastric ulcer disease
 
Radiological imaging of pleural diseases
Radiological imaging of pleural diseases Radiological imaging of pleural diseases
Radiological imaging of pleural diseases
 
Radiological imaging of mediastinal masses
Radiological imaging of mediastinal massesRadiological imaging of mediastinal masses
Radiological imaging of mediastinal masses
 
Ct chest type
Ct chest typeCt chest type
Ct chest type
 
Lung tumor radiology
Lung tumor radiologyLung tumor radiology
Lung tumor radiology
 
Acute Abdomen-Radiology
Acute Abdomen-RadiologyAcute Abdomen-Radiology
Acute Abdomen-Radiology
 
Barium swallow diseases
Barium swallow diseasesBarium swallow diseases
Barium swallow diseases
 
Radiology Spots PPT- 3 by Dr Chandni Wadhwani
 Radiology Spots PPT- 3 by Dr Chandni Wadhwani Radiology Spots PPT- 3 by Dr Chandni Wadhwani
Radiology Spots PPT- 3 by Dr Chandni Wadhwani
 
Signs in Chest Xray
Signs in Chest Xray Signs in Chest Xray
Signs in Chest Xray
 
Solitary pulmonary nodule
Solitary pulmonary noduleSolitary pulmonary nodule
Solitary pulmonary nodule
 
Imaging in Skull base
Imaging in Skull baseImaging in Skull base
Imaging in Skull base
 
Imaging of pulmonary embolism
Imaging of pulmonary embolismImaging of pulmonary embolism
Imaging of pulmonary embolism
 
Gastric carcinoma radiology ppt
Gastric carcinoma radiology  ppt Gastric carcinoma radiology  ppt
Gastric carcinoma radiology ppt
 
Ultrasound of spinal cord in neonates Dr. Muhammad Bin Zulfiqar
Ultrasound of spinal cord in neonates Dr. Muhammad Bin ZulfiqarUltrasound of spinal cord in neonates Dr. Muhammad Bin Zulfiqar
Ultrasound of spinal cord in neonates Dr. Muhammad Bin Zulfiqar
 
Presentation1.pptx, radiological signs in thoracic radiology.
Presentation1.pptx, radiological signs in thoracic radiology.Presentation1.pptx, radiological signs in thoracic radiology.
Presentation1.pptx, radiological signs in thoracic radiology.
 
INFLAMMATORY BOWEL DISEASE IMAGING(RADIOLOGY)
INFLAMMATORY BOWEL DISEASE IMAGING(RADIOLOGY)INFLAMMATORY BOWEL DISEASE IMAGING(RADIOLOGY)
INFLAMMATORY BOWEL DISEASE IMAGING(RADIOLOGY)
 
CT - Lung Carcinoma
CT - Lung CarcinomaCT - Lung Carcinoma
CT - Lung Carcinoma
 
Information regarding e lora for AERB registration
Information regarding e lora for AERB registrationInformation regarding e lora for AERB registration
Information regarding e lora for AERB registration
 

Destaque

Radiography questions + answers
Radiography questions + answersRadiography questions + answers
Radiography questions + answerspanshanger
 
Compilation of syndromes for last minute revision tips for neet pg, usmle , p...
Compilation of syndromes for last minute revision tips for neet pg, usmle , p...Compilation of syndromes for last minute revision tips for neet pg, usmle , p...
Compilation of syndromes for last minute revision tips for neet pg, usmle , p...Abhishek Gupta
 
500 single best answers in medicine
500 single best answers in medicine500 single best answers in medicine
500 single best answers in medicinehamadadodo
 
Cardiology Mnemonics
Cardiology MnemonicsCardiology Mnemonics
Cardiology MnemonicsDr UAK
 
Medical mnemonics pdf_print_version
Medical mnemonics pdf_print_versionMedical mnemonics pdf_print_version
Medical mnemonics pdf_print_version维卡普 Pardhu
 
Medico apps resources on clinical signs
Medico apps resources on clinical signsMedico apps resources on clinical signs
Medico apps resources on clinical signsAbhishek Gupta
 
Mcq 1060 questions
Mcq 1060 questionsMcq 1060 questions
Mcq 1060 questionsadrioz
 
basics of chest X- ray interpretation
basics of chest X- ray interpretationbasics of chest X- ray interpretation
basics of chest X- ray interpretationMaha Yousif
 
Endocrinology - the anterior pituitary gland
Endocrinology - the anterior pituitary glandEndocrinology - the anterior pituitary gland
Endocrinology - the anterior pituitary glandmeducationdotnet
 

Destaque (17)

Radiography questions + answers
Radiography questions + answersRadiography questions + answers
Radiography questions + answers
 
Compilation of syndromes for last minute revision tips for neet pg, usmle , p...
Compilation of syndromes for last minute revision tips for neet pg, usmle , p...Compilation of syndromes for last minute revision tips for neet pg, usmle , p...
Compilation of syndromes for last minute revision tips for neet pg, usmle , p...
 
Kamal FMGE MCQ Book
Kamal FMGE MCQ BookKamal FMGE MCQ Book
Kamal FMGE MCQ Book
 
Chest radiology quiz
Chest radiology quizChest radiology quiz
Chest radiology quiz
 
Physiology last-moment-revisions
Physiology last-moment-revisionsPhysiology last-moment-revisions
Physiology last-moment-revisions
 
Tips to Remember - fmge
Tips to Remember - fmgeTips to Remember - fmge
Tips to Remember - fmge
 
High Yield points surgery - mci screening test
High Yield points surgery - mci screening testHigh Yield points surgery - mci screening test
High Yield points surgery - mci screening test
 
500 single best answers in medicine
500 single best answers in medicine500 single best answers in medicine
500 single best answers in medicine
 
Pediatrics mnemonics
Pediatrics mnemonicsPediatrics mnemonics
Pediatrics mnemonics
 
Cardiology Mnemonics
Cardiology MnemonicsCardiology Mnemonics
Cardiology Mnemonics
 
Endocrine system mnemonics
Endocrine system mnemonicsEndocrine system mnemonics
Endocrine system mnemonics
 
GI system mnemonics
GI system mnemonicsGI system mnemonics
GI system mnemonics
 
Medical mnemonics pdf_print_version
Medical mnemonics pdf_print_versionMedical mnemonics pdf_print_version
Medical mnemonics pdf_print_version
 
Medico apps resources on clinical signs
Medico apps resources on clinical signsMedico apps resources on clinical signs
Medico apps resources on clinical signs
 
Mcq 1060 questions
Mcq 1060 questionsMcq 1060 questions
Mcq 1060 questions
 
basics of chest X- ray interpretation
basics of chest X- ray interpretationbasics of chest X- ray interpretation
basics of chest X- ray interpretation
 
Endocrinology - the anterior pituitary gland
Endocrinology - the anterior pituitary glandEndocrinology - the anterior pituitary gland
Endocrinology - the anterior pituitary gland
 

Semelhante a Radiology most important signs sample questions based on neet pg , usmle, plab and fmge pattern (mci screening)

23205036
2320503623205036
23205036radgirl
 
Previous year question on pneumothorax based on neet pg, usmle, plab and fmge...
Previous year question on pneumothorax based on neet pg, usmle, plab and fmge...Previous year question on pneumothorax based on neet pg, usmle, plab and fmge...
Previous year question on pneumothorax based on neet pg, usmle, plab and fmge...Abhishek Gupta
 
23204973
2320497323204973
23204973radgirl
 
23205045
2320504523205045
23205045radgirl
 
Role of Radiology in Pulmonary Tuberculosis
Role of Radiology in Pulmonary TuberculosisRole of Radiology in Pulmonary Tuberculosis
Role of Radiology in Pulmonary TuberculosisWaseem M.Nizamani
 
Role of Radiology in Pulmonary Tuberculosis
Role of Radiology in Pulmonary TuberculosisRole of Radiology in Pulmonary Tuberculosis
Role of Radiology in Pulmonary TuberculosisWaseem M.Nizamani
 
Previous year question on tonsils based on neet pg, usmle, plab and fmge or m...
Previous year question on tonsils based on neet pg, usmle, plab and fmge or m...Previous year question on tonsils based on neet pg, usmle, plab and fmge or m...
Previous year question on tonsils based on neet pg, usmle, plab and fmge or m...Abhishek Gupta
 
Abdominal Pain as Initial Presentation of Lung Adenocarcinoma
Abdominal Pain as Initial Presentation of Lung AdenocarcinomaAbdominal Pain as Initial Presentation of Lung Adenocarcinoma
Abdominal Pain as Initial Presentation of Lung Adenocarcinomaasclepiuspdfs
 
23205048
2320504823205048
23205048radgirl
 
23204946
2320494623204946
23204946radgirl
 
23205016
2320501623205016
23205016radgirl
 
23204961
2320496123204961
23204961radgirl
 
23204949
2320494923204949
23204949radgirl
 
Imaging of Pulmonary Embolism
Imaging of Pulmonary Embolism  Imaging of Pulmonary Embolism
Imaging of Pulmonary Embolism Gamal Agmy
 
Drs. Milam and Thomas's CMC X-Ray Mastery Project: May Cases
Drs. Milam and Thomas's CMC X-Ray Mastery Project: May CasesDrs. Milam and Thomas's CMC X-Ray Mastery Project: May Cases
Drs. Milam and Thomas's CMC X-Ray Mastery Project: May CasesSean M. Fox
 
Ultrasonography in Critically Ill Patients
Ultrasonography in Critically Ill PatientsUltrasonography in Critically Ill Patients
Ultrasonography in Critically Ill PatientsGamal Agmy
 
primary leiomyosarcoma of IVC: CCR Presented by Dr Anil Kumar.Senior Resident...
primary leiomyosarcoma of IVC: CCR Presented by Dr Anil Kumar.Senior Resident...primary leiomyosarcoma of IVC: CCR Presented by Dr Anil Kumar.Senior Resident...
primary leiomyosarcoma of IVC: CCR Presented by Dr Anil Kumar.Senior Resident...Anil Kumar
 
Dr Anil:AIIMS Patna, Primary leiomyosarcoma of IVC.
Dr Anil:AIIMS Patna, Primary leiomyosarcoma of IVC.Dr Anil:AIIMS Patna, Primary leiomyosarcoma of IVC.
Dr Anil:AIIMS Patna, Primary leiomyosarcoma of IVC.Anil Kumar
 
23204964
2320496423204964
23204964radgirl
 

Semelhante a Radiology most important signs sample questions based on neet pg , usmle, plab and fmge pattern (mci screening) (20)

23205036
2320503623205036
23205036
 
Previous year question on pneumothorax based on neet pg, usmle, plab and fmge...
Previous year question on pneumothorax based on neet pg, usmle, plab and fmge...Previous year question on pneumothorax based on neet pg, usmle, plab and fmge...
Previous year question on pneumothorax based on neet pg, usmle, plab and fmge...
 
23204973
2320497323204973
23204973
 
23205045
2320504523205045
23205045
 
Role of Radiology in Pulmonary Tuberculosis
Role of Radiology in Pulmonary TuberculosisRole of Radiology in Pulmonary Tuberculosis
Role of Radiology in Pulmonary Tuberculosis
 
Role of Radiology in Pulmonary Tuberculosis
Role of Radiology in Pulmonary TuberculosisRole of Radiology in Pulmonary Tuberculosis
Role of Radiology in Pulmonary Tuberculosis
 
Pe talk sep 21 dr.anjali
Pe talk sep 21 dr.anjaliPe talk sep 21 dr.anjali
Pe talk sep 21 dr.anjali
 
Previous year question on tonsils based on neet pg, usmle, plab and fmge or m...
Previous year question on tonsils based on neet pg, usmle, plab and fmge or m...Previous year question on tonsils based on neet pg, usmle, plab and fmge or m...
Previous year question on tonsils based on neet pg, usmle, plab and fmge or m...
 
Abdominal Pain as Initial Presentation of Lung Adenocarcinoma
Abdominal Pain as Initial Presentation of Lung AdenocarcinomaAbdominal Pain as Initial Presentation of Lung Adenocarcinoma
Abdominal Pain as Initial Presentation of Lung Adenocarcinoma
 
23205048
2320504823205048
23205048
 
23204946
2320494623204946
23204946
 
23205016
2320501623205016
23205016
 
23204961
2320496123204961
23204961
 
23204949
2320494923204949
23204949
 
Imaging of Pulmonary Embolism
Imaging of Pulmonary Embolism  Imaging of Pulmonary Embolism
Imaging of Pulmonary Embolism
 
Drs. Milam and Thomas's CMC X-Ray Mastery Project: May Cases
Drs. Milam and Thomas's CMC X-Ray Mastery Project: May CasesDrs. Milam and Thomas's CMC X-Ray Mastery Project: May Cases
Drs. Milam and Thomas's CMC X-Ray Mastery Project: May Cases
 
Ultrasonography in Critically Ill Patients
Ultrasonography in Critically Ill PatientsUltrasonography in Critically Ill Patients
Ultrasonography in Critically Ill Patients
 
primary leiomyosarcoma of IVC: CCR Presented by Dr Anil Kumar.Senior Resident...
primary leiomyosarcoma of IVC: CCR Presented by Dr Anil Kumar.Senior Resident...primary leiomyosarcoma of IVC: CCR Presented by Dr Anil Kumar.Senior Resident...
primary leiomyosarcoma of IVC: CCR Presented by Dr Anil Kumar.Senior Resident...
 
Dr Anil:AIIMS Patna, Primary leiomyosarcoma of IVC.
Dr Anil:AIIMS Patna, Primary leiomyosarcoma of IVC.Dr Anil:AIIMS Patna, Primary leiomyosarcoma of IVC.
Dr Anil:AIIMS Patna, Primary leiomyosarcoma of IVC.
 
23204964
2320496423204964
23204964
 

Mais de Medico Apps

Previous year question on hepatomegaly based on neet pg, usmle, plab and fmge...
Previous year question on hepatomegaly based on neet pg, usmle, plab and fmge...Previous year question on hepatomegaly based on neet pg, usmle, plab and fmge...
Previous year question on hepatomegaly based on neet pg, usmle, plab and fmge...Medico Apps
 
Previous year question on rabies based on neet pg, usmle, plab and fmge or mc...
Previous year question on rabies based on neet pg, usmle, plab and fmge or mc...Previous year question on rabies based on neet pg, usmle, plab and fmge or mc...
Previous year question on rabies based on neet pg, usmle, plab and fmge or mc...Medico Apps
 
Previous year question on medulloblastoma based on neet pg, usmle, plab and f...
Previous year question on medulloblastoma based on neet pg, usmle, plab and f...Previous year question on medulloblastoma based on neet pg, usmle, plab and f...
Previous year question on medulloblastoma based on neet pg, usmle, plab and f...Medico Apps
 
Previous year question on cataract based on neet pg, usmle, plab and fmge or ...
Previous year question on cataract based on neet pg, usmle, plab and fmge or ...Previous year question on cataract based on neet pg, usmle, plab and fmge or ...
Previous year question on cataract based on neet pg, usmle, plab and fmge or ...Medico Apps
 
Previous year question on pharyngeal arches embryology based on neet pg, usml...
Previous year question on pharyngeal arches embryology based on neet pg, usml...Previous year question on pharyngeal arches embryology based on neet pg, usml...
Previous year question on pharyngeal arches embryology based on neet pg, usml...Medico Apps
 
Previous year question on pemphigus vulgaris based on neet pg, usmle, plab an...
Previous year question on pemphigus vulgaris based on neet pg, usmle, plab an...Previous year question on pemphigus vulgaris based on neet pg, usmle, plab an...
Previous year question on pemphigus vulgaris based on neet pg, usmle, plab an...Medico Apps
 
Previous year question on leptospirosis based on neet pg, usmle, plab and fmg...
Previous year question on leptospirosis based on neet pg, usmle, plab and fmg...Previous year question on leptospirosis based on neet pg, usmle, plab and fmg...
Previous year question on leptospirosis based on neet pg, usmle, plab and fmg...Medico Apps
 
Previous year question on glycolysis based on neet pg, usmle, plab and fmge o...
Previous year question on glycolysis based on neet pg, usmle, plab and fmge o...Previous year question on glycolysis based on neet pg, usmle, plab and fmge o...
Previous year question on glycolysis based on neet pg, usmle, plab and fmge o...Medico Apps
 
Previous year question on glycolysis based on neet pg, usmle, plab and fmge o...
Previous year question on glycolysis based on neet pg, usmle, plab and fmge o...Previous year question on glycolysis based on neet pg, usmle, plab and fmge o...
Previous year question on glycolysis based on neet pg, usmle, plab and fmge o...Medico Apps
 
Previous year question on bone cyst based on neet pg, usmle, plab and fmge or...
Previous year question on bone cyst based on neet pg, usmle, plab and fmge or...Previous year question on bone cyst based on neet pg, usmle, plab and fmge or...
Previous year question on bone cyst based on neet pg, usmle, plab and fmge or...Medico Apps
 
Previous year question on ketamine based on neet pg, usmle, plab and fmge or ...
Previous year question on ketamine based on neet pg, usmle, plab and fmge or ...Previous year question on ketamine based on neet pg, usmle, plab and fmge or ...
Previous year question on ketamine based on neet pg, usmle, plab and fmge or ...Medico Apps
 
Ketamine (anaesthesia )sample questions based on neet pg , usmle, plab and fm...
Ketamine (anaesthesia )sample questions based on neet pg , usmle, plab and fm...Ketamine (anaesthesia )sample questions based on neet pg , usmle, plab and fm...
Ketamine (anaesthesia )sample questions based on neet pg , usmle, plab and fm...Medico Apps
 
Hiv aids sample questions based on neet pg , usmle, plab and fmge pattern (mc...
Hiv aids sample questions based on neet pg , usmle, plab and fmge pattern (mc...Hiv aids sample questions based on neet pg , usmle, plab and fmge pattern (mc...
Hiv aids sample questions based on neet pg , usmle, plab and fmge pattern (mc...Medico Apps
 
Hiv aids sample questions based on neet pg , usmle, plab and fmge pattern (mc...
Hiv aids sample questions based on neet pg , usmle, plab and fmge pattern (mc...Hiv aids sample questions based on neet pg , usmle, plab and fmge pattern (mc...
Hiv aids sample questions based on neet pg , usmle, plab and fmge pattern (mc...Medico Apps
 
Hiv aids sample questions based on neet pg , usmle, plab and fmge pattern (mc...
Hiv aids sample questions based on neet pg , usmle, plab and fmge pattern (mc...Hiv aids sample questions based on neet pg , usmle, plab and fmge pattern (mc...
Hiv aids sample questions based on neet pg , usmle, plab and fmge pattern (mc...Medico Apps
 
Hiv aids sample questions based on neet pg , usmle, plab and fmge pattern (mc...
Hiv aids sample questions based on neet pg , usmle, plab and fmge pattern (mc...Hiv aids sample questions based on neet pg , usmle, plab and fmge pattern (mc...
Hiv aids sample questions based on neet pg , usmle, plab and fmge pattern (mc...Medico Apps
 
Hiv aids sample questions based on neet pg , usmle, plab and fmge pattern (mc...
Hiv aids sample questions based on neet pg , usmle, plab and fmge pattern (mc...Hiv aids sample questions based on neet pg , usmle, plab and fmge pattern (mc...
Hiv aids sample questions based on neet pg , usmle, plab and fmge pattern (mc...Medico Apps
 
Meconium stained baby sample questions based on neet pg , usmle, plab and fmg...
Meconium stained baby sample questions based on neet pg , usmle, plab and fmg...Meconium stained baby sample questions based on neet pg , usmle, plab and fmg...
Meconium stained baby sample questions based on neet pg , usmle, plab and fmg...Medico Apps
 
Pancreatitis sample questions based on neet pg , usmle, plab and fmge pattern...
Pancreatitis sample questions based on neet pg , usmle, plab and fmge pattern...Pancreatitis sample questions based on neet pg , usmle, plab and fmge pattern...
Pancreatitis sample questions based on neet pg , usmle, plab and fmge pattern...Medico Apps
 
Alzheimers disease sample questions based on neet pg , usmle, plab and fmge p...
Alzheimers disease sample questions based on neet pg , usmle, plab and fmge p...Alzheimers disease sample questions based on neet pg , usmle, plab and fmge p...
Alzheimers disease sample questions based on neet pg , usmle, plab and fmge p...Medico Apps
 

Mais de Medico Apps (20)

Previous year question on hepatomegaly based on neet pg, usmle, plab and fmge...
Previous year question on hepatomegaly based on neet pg, usmle, plab and fmge...Previous year question on hepatomegaly based on neet pg, usmle, plab and fmge...
Previous year question on hepatomegaly based on neet pg, usmle, plab and fmge...
 
Previous year question on rabies based on neet pg, usmle, plab and fmge or mc...
Previous year question on rabies based on neet pg, usmle, plab and fmge or mc...Previous year question on rabies based on neet pg, usmle, plab and fmge or mc...
Previous year question on rabies based on neet pg, usmle, plab and fmge or mc...
 
Previous year question on medulloblastoma based on neet pg, usmle, plab and f...
Previous year question on medulloblastoma based on neet pg, usmle, plab and f...Previous year question on medulloblastoma based on neet pg, usmle, plab and f...
Previous year question on medulloblastoma based on neet pg, usmle, plab and f...
 
Previous year question on cataract based on neet pg, usmle, plab and fmge or ...
Previous year question on cataract based on neet pg, usmle, plab and fmge or ...Previous year question on cataract based on neet pg, usmle, plab and fmge or ...
Previous year question on cataract based on neet pg, usmle, plab and fmge or ...
 
Previous year question on pharyngeal arches embryology based on neet pg, usml...
Previous year question on pharyngeal arches embryology based on neet pg, usml...Previous year question on pharyngeal arches embryology based on neet pg, usml...
Previous year question on pharyngeal arches embryology based on neet pg, usml...
 
Previous year question on pemphigus vulgaris based on neet pg, usmle, plab an...
Previous year question on pemphigus vulgaris based on neet pg, usmle, plab an...Previous year question on pemphigus vulgaris based on neet pg, usmle, plab an...
Previous year question on pemphigus vulgaris based on neet pg, usmle, plab an...
 
Previous year question on leptospirosis based on neet pg, usmle, plab and fmg...
Previous year question on leptospirosis based on neet pg, usmle, plab and fmg...Previous year question on leptospirosis based on neet pg, usmle, plab and fmg...
Previous year question on leptospirosis based on neet pg, usmle, plab and fmg...
 
Previous year question on glycolysis based on neet pg, usmle, plab and fmge o...
Previous year question on glycolysis based on neet pg, usmle, plab and fmge o...Previous year question on glycolysis based on neet pg, usmle, plab and fmge o...
Previous year question on glycolysis based on neet pg, usmle, plab and fmge o...
 
Previous year question on glycolysis based on neet pg, usmle, plab and fmge o...
Previous year question on glycolysis based on neet pg, usmle, plab and fmge o...Previous year question on glycolysis based on neet pg, usmle, plab and fmge o...
Previous year question on glycolysis based on neet pg, usmle, plab and fmge o...
 
Previous year question on bone cyst based on neet pg, usmle, plab and fmge or...
Previous year question on bone cyst based on neet pg, usmle, plab and fmge or...Previous year question on bone cyst based on neet pg, usmle, plab and fmge or...
Previous year question on bone cyst based on neet pg, usmle, plab and fmge or...
 
Previous year question on ketamine based on neet pg, usmle, plab and fmge or ...
Previous year question on ketamine based on neet pg, usmle, plab and fmge or ...Previous year question on ketamine based on neet pg, usmle, plab and fmge or ...
Previous year question on ketamine based on neet pg, usmle, plab and fmge or ...
 
Ketamine (anaesthesia )sample questions based on neet pg , usmle, plab and fm...
Ketamine (anaesthesia )sample questions based on neet pg , usmle, plab and fm...Ketamine (anaesthesia )sample questions based on neet pg , usmle, plab and fm...
Ketamine (anaesthesia )sample questions based on neet pg , usmle, plab and fm...
 
Hiv aids sample questions based on neet pg , usmle, plab and fmge pattern (mc...
Hiv aids sample questions based on neet pg , usmle, plab and fmge pattern (mc...Hiv aids sample questions based on neet pg , usmle, plab and fmge pattern (mc...
Hiv aids sample questions based on neet pg , usmle, plab and fmge pattern (mc...
 
Hiv aids sample questions based on neet pg , usmle, plab and fmge pattern (mc...
Hiv aids sample questions based on neet pg , usmle, plab and fmge pattern (mc...Hiv aids sample questions based on neet pg , usmle, plab and fmge pattern (mc...
Hiv aids sample questions based on neet pg , usmle, plab and fmge pattern (mc...
 
Hiv aids sample questions based on neet pg , usmle, plab and fmge pattern (mc...
Hiv aids sample questions based on neet pg , usmle, plab and fmge pattern (mc...Hiv aids sample questions based on neet pg , usmle, plab and fmge pattern (mc...
Hiv aids sample questions based on neet pg , usmle, plab and fmge pattern (mc...
 
Hiv aids sample questions based on neet pg , usmle, plab and fmge pattern (mc...
Hiv aids sample questions based on neet pg , usmle, plab and fmge pattern (mc...Hiv aids sample questions based on neet pg , usmle, plab and fmge pattern (mc...
Hiv aids sample questions based on neet pg , usmle, plab and fmge pattern (mc...
 
Hiv aids sample questions based on neet pg , usmle, plab and fmge pattern (mc...
Hiv aids sample questions based on neet pg , usmle, plab and fmge pattern (mc...Hiv aids sample questions based on neet pg , usmle, plab and fmge pattern (mc...
Hiv aids sample questions based on neet pg , usmle, plab and fmge pattern (mc...
 
Meconium stained baby sample questions based on neet pg , usmle, plab and fmg...
Meconium stained baby sample questions based on neet pg , usmle, plab and fmg...Meconium stained baby sample questions based on neet pg , usmle, plab and fmg...
Meconium stained baby sample questions based on neet pg , usmle, plab and fmg...
 
Pancreatitis sample questions based on neet pg , usmle, plab and fmge pattern...
Pancreatitis sample questions based on neet pg , usmle, plab and fmge pattern...Pancreatitis sample questions based on neet pg , usmle, plab and fmge pattern...
Pancreatitis sample questions based on neet pg , usmle, plab and fmge pattern...
 
Alzheimers disease sample questions based on neet pg , usmle, plab and fmge p...
Alzheimers disease sample questions based on neet pg , usmle, plab and fmge p...Alzheimers disease sample questions based on neet pg , usmle, plab and fmge p...
Alzheimers disease sample questions based on neet pg , usmle, plab and fmge p...
 

Último

Best Rate (Hyderabad) Call Girls Jahanuma ⟟ 8250192130 ⟟ High Class Call Girl...
Best Rate (Hyderabad) Call Girls Jahanuma ⟟ 8250192130 ⟟ High Class Call Girl...Best Rate (Hyderabad) Call Girls Jahanuma ⟟ 8250192130 ⟟ High Class Call Girl...
Best Rate (Hyderabad) Call Girls Jahanuma ⟟ 8250192130 ⟟ High Class Call Girl...astropune
 
Book Paid Powai Call Girls Mumbai 𖠋 9930245274 𖠋Low Budget Full Independent H...
Book Paid Powai Call Girls Mumbai 𖠋 9930245274 𖠋Low Budget Full Independent H...Book Paid Powai Call Girls Mumbai 𖠋 9930245274 𖠋Low Budget Full Independent H...
Book Paid Powai Call Girls Mumbai 𖠋 9930245274 𖠋Low Budget Full Independent H...Call Girls in Nagpur High Profile
 
Call Girls Ludhiana Just Call 9907093804 Top Class Call Girl Service Available
Call Girls Ludhiana Just Call 9907093804 Top Class Call Girl Service AvailableCall Girls Ludhiana Just Call 9907093804 Top Class Call Girl Service Available
Call Girls Ludhiana Just Call 9907093804 Top Class Call Girl Service AvailableDipal Arora
 
Pondicherry Call Girls Book Now 9630942363 Top Class Pondicherry Escort Servi...
Pondicherry Call Girls Book Now 9630942363 Top Class Pondicherry Escort Servi...Pondicherry Call Girls Book Now 9630942363 Top Class Pondicherry Escort Servi...
Pondicherry Call Girls Book Now 9630942363 Top Class Pondicherry Escort Servi...Genuine Call Girls
 
Lucknow Call girls - 8800925952 - 24x7 service with hotel room
Lucknow Call girls - 8800925952 - 24x7 service with hotel roomLucknow Call girls - 8800925952 - 24x7 service with hotel room
Lucknow Call girls - 8800925952 - 24x7 service with hotel roomdiscovermytutordmt
 
All Time Service Available Call Girls Marine Drive 📳 9820252231 For 18+ VIP C...
All Time Service Available Call Girls Marine Drive 📳 9820252231 For 18+ VIP C...All Time Service Available Call Girls Marine Drive 📳 9820252231 For 18+ VIP C...
All Time Service Available Call Girls Marine Drive 📳 9820252231 For 18+ VIP C...Arohi Goyal
 
Call Girls Gwalior Just Call 9907093804 Top Class Call Girl Service Available
Call Girls Gwalior Just Call 9907093804 Top Class Call Girl Service AvailableCall Girls Gwalior Just Call 9907093804 Top Class Call Girl Service Available
Call Girls Gwalior Just Call 9907093804 Top Class Call Girl Service AvailableDipal Arora
 
Russian Escorts Girls Nehru Place ZINATHI 🔝9711199012 ☪ 24/7 Call Girls Delhi
Russian Escorts Girls  Nehru Place ZINATHI 🔝9711199012 ☪ 24/7 Call Girls DelhiRussian Escorts Girls  Nehru Place ZINATHI 🔝9711199012 ☪ 24/7 Call Girls Delhi
Russian Escorts Girls Nehru Place ZINATHI 🔝9711199012 ☪ 24/7 Call Girls DelhiAlinaDevecerski
 
Call Girls Kochi Just Call 9907093804 Top Class Call Girl Service Available
Call Girls Kochi Just Call 9907093804 Top Class Call Girl Service AvailableCall Girls Kochi Just Call 9907093804 Top Class Call Girl Service Available
Call Girls Kochi Just Call 9907093804 Top Class Call Girl Service AvailableDipal Arora
 
Call Girls Horamavu WhatsApp Number 7001035870 Meeting With Bangalore Escorts
Call Girls Horamavu WhatsApp Number 7001035870 Meeting With Bangalore EscortsCall Girls Horamavu WhatsApp Number 7001035870 Meeting With Bangalore Escorts
Call Girls Horamavu WhatsApp Number 7001035870 Meeting With Bangalore Escortsvidya singh
 
♛VVIP Hyderabad Call Girls Chintalkunta🖕7001035870🖕Riya Kappor Top Call Girl ...
♛VVIP Hyderabad Call Girls Chintalkunta🖕7001035870🖕Riya Kappor Top Call Girl ...♛VVIP Hyderabad Call Girls Chintalkunta🖕7001035870🖕Riya Kappor Top Call Girl ...
♛VVIP Hyderabad Call Girls Chintalkunta🖕7001035870🖕Riya Kappor Top Call Girl ...astropune
 
Top Rated Bangalore Call Girls Mg Road ⟟ 9332606886 ⟟ Call Me For Genuine S...
Top Rated Bangalore Call Girls Mg Road ⟟   9332606886 ⟟ Call Me For Genuine S...Top Rated Bangalore Call Girls Mg Road ⟟   9332606886 ⟟ Call Me For Genuine S...
Top Rated Bangalore Call Girls Mg Road ⟟ 9332606886 ⟟ Call Me For Genuine S...narwatsonia7
 
Night 7k to 12k Navi Mumbai Call Girl Photo 👉 BOOK NOW 9833363713 👈 ♀️ night ...
Night 7k to 12k Navi Mumbai Call Girl Photo 👉 BOOK NOW 9833363713 👈 ♀️ night ...Night 7k to 12k Navi Mumbai Call Girl Photo 👉 BOOK NOW 9833363713 👈 ♀️ night ...
Night 7k to 12k Navi Mumbai Call Girl Photo 👉 BOOK NOW 9833363713 👈 ♀️ night ...aartirawatdelhi
 
Premium Bangalore Call Girls Jigani Dail 6378878445 Escort Service For Hot Ma...
Premium Bangalore Call Girls Jigani Dail 6378878445 Escort Service For Hot Ma...Premium Bangalore Call Girls Jigani Dail 6378878445 Escort Service For Hot Ma...
Premium Bangalore Call Girls Jigani Dail 6378878445 Escort Service For Hot Ma...tanya dube
 
Call Girls Haridwar Just Call 9907093804 Top Class Call Girl Service Available
Call Girls Haridwar Just Call 9907093804 Top Class Call Girl Service AvailableCall Girls Haridwar Just Call 9907093804 Top Class Call Girl Service Available
Call Girls Haridwar Just Call 9907093804 Top Class Call Girl Service AvailableDipal Arora
 
VIP Service Call Girls Sindhi Colony 📳 7877925207 For 18+ VIP Call Girl At Th...
VIP Service Call Girls Sindhi Colony 📳 7877925207 For 18+ VIP Call Girl At Th...VIP Service Call Girls Sindhi Colony 📳 7877925207 For 18+ VIP Call Girl At Th...
VIP Service Call Girls Sindhi Colony 📳 7877925207 For 18+ VIP Call Girl At Th...jageshsingh5554
 
Best Rate (Guwahati ) Call Girls Guwahati ⟟ 8617370543 ⟟ High Class Call Girl...
Best Rate (Guwahati ) Call Girls Guwahati ⟟ 8617370543 ⟟ High Class Call Girl...Best Rate (Guwahati ) Call Girls Guwahati ⟟ 8617370543 ⟟ High Class Call Girl...
Best Rate (Guwahati ) Call Girls Guwahati ⟟ 8617370543 ⟟ High Class Call Girl...Dipal Arora
 
Call Girls Aurangabad Just Call 9907093804 Top Class Call Girl Service Available
Call Girls Aurangabad Just Call 9907093804 Top Class Call Girl Service AvailableCall Girls Aurangabad Just Call 9907093804 Top Class Call Girl Service Available
Call Girls Aurangabad Just Call 9907093804 Top Class Call Girl Service AvailableDipal Arora
 
Call Girls Varanasi Just Call 9907093804 Top Class Call Girl Service Available
Call Girls Varanasi Just Call 9907093804 Top Class Call Girl Service AvailableCall Girls Varanasi Just Call 9907093804 Top Class Call Girl Service Available
Call Girls Varanasi Just Call 9907093804 Top Class Call Girl Service AvailableDipal Arora
 
Top Quality Call Girl Service Kalyanpur 6378878445 Available Call Girls Any Time
Top Quality Call Girl Service Kalyanpur 6378878445 Available Call Girls Any TimeTop Quality Call Girl Service Kalyanpur 6378878445 Available Call Girls Any Time
Top Quality Call Girl Service Kalyanpur 6378878445 Available Call Girls Any TimeCall Girls Delhi
 

Último (20)

Best Rate (Hyderabad) Call Girls Jahanuma ⟟ 8250192130 ⟟ High Class Call Girl...
Best Rate (Hyderabad) Call Girls Jahanuma ⟟ 8250192130 ⟟ High Class Call Girl...Best Rate (Hyderabad) Call Girls Jahanuma ⟟ 8250192130 ⟟ High Class Call Girl...
Best Rate (Hyderabad) Call Girls Jahanuma ⟟ 8250192130 ⟟ High Class Call Girl...
 
Book Paid Powai Call Girls Mumbai 𖠋 9930245274 𖠋Low Budget Full Independent H...
Book Paid Powai Call Girls Mumbai 𖠋 9930245274 𖠋Low Budget Full Independent H...Book Paid Powai Call Girls Mumbai 𖠋 9930245274 𖠋Low Budget Full Independent H...
Book Paid Powai Call Girls Mumbai 𖠋 9930245274 𖠋Low Budget Full Independent H...
 
Call Girls Ludhiana Just Call 9907093804 Top Class Call Girl Service Available
Call Girls Ludhiana Just Call 9907093804 Top Class Call Girl Service AvailableCall Girls Ludhiana Just Call 9907093804 Top Class Call Girl Service Available
Call Girls Ludhiana Just Call 9907093804 Top Class Call Girl Service Available
 
Pondicherry Call Girls Book Now 9630942363 Top Class Pondicherry Escort Servi...
Pondicherry Call Girls Book Now 9630942363 Top Class Pondicherry Escort Servi...Pondicherry Call Girls Book Now 9630942363 Top Class Pondicherry Escort Servi...
Pondicherry Call Girls Book Now 9630942363 Top Class Pondicherry Escort Servi...
 
Lucknow Call girls - 8800925952 - 24x7 service with hotel room
Lucknow Call girls - 8800925952 - 24x7 service with hotel roomLucknow Call girls - 8800925952 - 24x7 service with hotel room
Lucknow Call girls - 8800925952 - 24x7 service with hotel room
 
All Time Service Available Call Girls Marine Drive 📳 9820252231 For 18+ VIP C...
All Time Service Available Call Girls Marine Drive 📳 9820252231 For 18+ VIP C...All Time Service Available Call Girls Marine Drive 📳 9820252231 For 18+ VIP C...
All Time Service Available Call Girls Marine Drive 📳 9820252231 For 18+ VIP C...
 
Call Girls Gwalior Just Call 9907093804 Top Class Call Girl Service Available
Call Girls Gwalior Just Call 9907093804 Top Class Call Girl Service AvailableCall Girls Gwalior Just Call 9907093804 Top Class Call Girl Service Available
Call Girls Gwalior Just Call 9907093804 Top Class Call Girl Service Available
 
Russian Escorts Girls Nehru Place ZINATHI 🔝9711199012 ☪ 24/7 Call Girls Delhi
Russian Escorts Girls  Nehru Place ZINATHI 🔝9711199012 ☪ 24/7 Call Girls DelhiRussian Escorts Girls  Nehru Place ZINATHI 🔝9711199012 ☪ 24/7 Call Girls Delhi
Russian Escorts Girls Nehru Place ZINATHI 🔝9711199012 ☪ 24/7 Call Girls Delhi
 
Call Girls Kochi Just Call 9907093804 Top Class Call Girl Service Available
Call Girls Kochi Just Call 9907093804 Top Class Call Girl Service AvailableCall Girls Kochi Just Call 9907093804 Top Class Call Girl Service Available
Call Girls Kochi Just Call 9907093804 Top Class Call Girl Service Available
 
Call Girls Horamavu WhatsApp Number 7001035870 Meeting With Bangalore Escorts
Call Girls Horamavu WhatsApp Number 7001035870 Meeting With Bangalore EscortsCall Girls Horamavu WhatsApp Number 7001035870 Meeting With Bangalore Escorts
Call Girls Horamavu WhatsApp Number 7001035870 Meeting With Bangalore Escorts
 
♛VVIP Hyderabad Call Girls Chintalkunta🖕7001035870🖕Riya Kappor Top Call Girl ...
♛VVIP Hyderabad Call Girls Chintalkunta🖕7001035870🖕Riya Kappor Top Call Girl ...♛VVIP Hyderabad Call Girls Chintalkunta🖕7001035870🖕Riya Kappor Top Call Girl ...
♛VVIP Hyderabad Call Girls Chintalkunta🖕7001035870🖕Riya Kappor Top Call Girl ...
 
Top Rated Bangalore Call Girls Mg Road ⟟ 9332606886 ⟟ Call Me For Genuine S...
Top Rated Bangalore Call Girls Mg Road ⟟   9332606886 ⟟ Call Me For Genuine S...Top Rated Bangalore Call Girls Mg Road ⟟   9332606886 ⟟ Call Me For Genuine S...
Top Rated Bangalore Call Girls Mg Road ⟟ 9332606886 ⟟ Call Me For Genuine S...
 
Night 7k to 12k Navi Mumbai Call Girl Photo 👉 BOOK NOW 9833363713 👈 ♀️ night ...
Night 7k to 12k Navi Mumbai Call Girl Photo 👉 BOOK NOW 9833363713 👈 ♀️ night ...Night 7k to 12k Navi Mumbai Call Girl Photo 👉 BOOK NOW 9833363713 👈 ♀️ night ...
Night 7k to 12k Navi Mumbai Call Girl Photo 👉 BOOK NOW 9833363713 👈 ♀️ night ...
 
Premium Bangalore Call Girls Jigani Dail 6378878445 Escort Service For Hot Ma...
Premium Bangalore Call Girls Jigani Dail 6378878445 Escort Service For Hot Ma...Premium Bangalore Call Girls Jigani Dail 6378878445 Escort Service For Hot Ma...
Premium Bangalore Call Girls Jigani Dail 6378878445 Escort Service For Hot Ma...
 
Call Girls Haridwar Just Call 9907093804 Top Class Call Girl Service Available
Call Girls Haridwar Just Call 9907093804 Top Class Call Girl Service AvailableCall Girls Haridwar Just Call 9907093804 Top Class Call Girl Service Available
Call Girls Haridwar Just Call 9907093804 Top Class Call Girl Service Available
 
VIP Service Call Girls Sindhi Colony 📳 7877925207 For 18+ VIP Call Girl At Th...
VIP Service Call Girls Sindhi Colony 📳 7877925207 For 18+ VIP Call Girl At Th...VIP Service Call Girls Sindhi Colony 📳 7877925207 For 18+ VIP Call Girl At Th...
VIP Service Call Girls Sindhi Colony 📳 7877925207 For 18+ VIP Call Girl At Th...
 
Best Rate (Guwahati ) Call Girls Guwahati ⟟ 8617370543 ⟟ High Class Call Girl...
Best Rate (Guwahati ) Call Girls Guwahati ⟟ 8617370543 ⟟ High Class Call Girl...Best Rate (Guwahati ) Call Girls Guwahati ⟟ 8617370543 ⟟ High Class Call Girl...
Best Rate (Guwahati ) Call Girls Guwahati ⟟ 8617370543 ⟟ High Class Call Girl...
 
Call Girls Aurangabad Just Call 9907093804 Top Class Call Girl Service Available
Call Girls Aurangabad Just Call 9907093804 Top Class Call Girl Service AvailableCall Girls Aurangabad Just Call 9907093804 Top Class Call Girl Service Available
Call Girls Aurangabad Just Call 9907093804 Top Class Call Girl Service Available
 
Call Girls Varanasi Just Call 9907093804 Top Class Call Girl Service Available
Call Girls Varanasi Just Call 9907093804 Top Class Call Girl Service AvailableCall Girls Varanasi Just Call 9907093804 Top Class Call Girl Service Available
Call Girls Varanasi Just Call 9907093804 Top Class Call Girl Service Available
 
Top Quality Call Girl Service Kalyanpur 6378878445 Available Call Girls Any Time
Top Quality Call Girl Service Kalyanpur 6378878445 Available Call Girls Any TimeTop Quality Call Girl Service Kalyanpur 6378878445 Available Call Girls Any Time
Top Quality Call Girl Service Kalyanpur 6378878445 Available Call Girls Any Time
 

Radiology most important signs sample questions based on neet pg , usmle, plab and fmge pattern (mci screening)

  • 1. Earliest radiological sign of Pulmonary Venous Hypertension in Chest X-ray is: A: Cephalization of pulmonary vascularity B: Pleural effusion C: Kerley B lines D: Alveolar pulmonary edema Correct Ans:A Explanation The first radiographic sign of pulmonary venous hypertension is cephalization of the pulmonary vessels due to pulmonary vein and artery dilatation. In the normal individual, there is continuous passage of fluid from the pulmonary veins into adjacent interlobular lymphatics that return the fluid to the central mediastinal veins. If the lympatic reserve is overcome by increased transudate as a result of elevated pulmonary venous pressure, the interlobular septa are thickened and become visible radiographically. The first radiologic sign of pulmonary venous hypertension is cephalization of the pulmonary vessels. Ref: Atlas of Pulmonary Vascular Imaging By Conard Wittram, Page 51; Textbook of Radiology and Imaging By David Sutton, 5th Edition, Pages 288-89 Sample Previous Year Question on Radiological Signs based on previous Year Questions of NEET PG, USMLE,PLAB,FMGE (MCI Screening). Please visit www.medicoapps.org for more such Quizzes A 8 year old boy is being treated for rickets. Which of the following investigations shows the earliest evidence for healing? A: Serum Ca B: Serum phosphates C: Radiological examination of long bones D: Serum ALP Correct Ans:C Explanation The earliest feature of healing in rickets is seen in X ray of growing end of the bones.
  • 2. Sample Previous Year Question on Radiological Signs based on previous Year Questions of NEET PG, USMLE,PLAB,FMGE (MCI Screening). Please visit www.medicoapps.org for more such Quizzes Allergic reactions to radiological contrast agents are: A: Anaphylactic reactions B: IgE mediated reactions C: Urticaria D: Edema Correct Ans:A Explanation In radiological imaging it is usually believed that the adverse effects of contrast media are non-IgE mediated anaphylactoid reactions, as a result of released vasoactive amines (histamine, serotonin, etc) which trigger the mast cells directly. Since anaphylactoid reaction is not given in the choice, anaphylactic reaction may be the right choice. Ref: Encyclopedia of Diagnostic Imaging By Albert L Baert Pages 44-46 Sample Previous Year Question on Radiological Signs based on previous Year Questions of NEET PG, USMLE,PLAB,FMGE (MCI Screening). Please visit www.medicoapps.org for more such Quizzes Which radiological procedure is used for studying vesico-ureteric reflux? A: Ascending pyelogram B: Cystogra m C: Intravenous urogram D: Micturition cystourethrogram Correct Ans:D Explanation The diagnosis of vesico ureteric reflux is made using micturition cystourethrogram. Other techniques used in diagnosing vesico ureteric reflux are simple or delayed cystography or voiding cinefluoroscopy. In a case of vesico ureteral reflux, cystogram may show one of the following findings: Persistently dilated lower ureter, areas of dilatation in the ureter, ureter visualized throughout its entire length shows, presence of hydroureteronephrosis with a narrow juxtavesical ureteral segment or changes of healed pyelonephritis.
  • 3. Ref: Smith's General Urology, 17e, Chapter 12 Sample Previous Year Question on Radiological Signs based on previous Year Questions of NEET PG, USMLE,PLAB,FMGE (MCI Screening). Please visit www.medicoapps.org for more such Quizzes Radiological appearance of ‘thimble’ bladder is seen in the following condition: A: Cystitis cystica B: Chronic tuberculous cystitis C: Neurogenic bladder D: Acute tuberculous cystitis Correct Ans:B Explanation Early tuberculosis of the bladder commences around the ureteric orifice or trigone, the earliest evidence being pallor of the mucosa due to submucosal oedema. Subsequently, tubercles may be seen and, in long standing cases, there is marked fibrosis and the capacity of the bladder is greatly reduced giving the radiological appearance of ‘thimble bladder’. Ref: Bailey & Love's Short Practice of Surgery 26E edited by Norman Williams, Christopher Bulstrode, P Ronan O'Connell, 2013, Page 1327-1328 Sample Previous Year Question on Radiological Signs based on previous Year Questions of NEET PG, USMLE,PLAB,FMGE (MCI Screening). Please visit www.medicoapps.org for more such Quizzes Routine radiological examination of a middle aged man shows "Spongy appearance" with central sunburst calcification. This is seen in: A: Pancreatic adenocarcinoma B: Mucinous cystadenocarcinoma C: Somatostatinoma D: Serous cystadenoma Correct Ans:D Explanation
  • 4. Serous cystadenoma is a rare solitary, benign cystic neoplasm of the pancreas. It consists of multiple small locules lined by cuboidal epithelium, the cells of which contain abundant glycogen (also called microcystic, serous, and glycogen-rich cystadenoma). Radiological appearance of serous cystadenoma of pancreas shows characteristic central sunburst calcificationwhich is pathognomonic. Ref: The Pancreas: An Integrated Textbook of Basic Science, Medicine, and Surgery edited by Hans-Gunther Beger, Markus Buchler, Richard Kozarek, Markus Lerch, John Neoptolemos, Andrew Warshaw, David Whitcomb, Keiko Shiratori, 2009, Page 933. Sample Previous Year Question on Radiological Signs based on previous Year Questions of NEET PG, USMLE,PLAB,FMGE (MCI Screening). Please visit www.medicoapps.org for more such Quizzes Which of the following is the radiological finding of a benign gastric ulcer? A: Hampton line B: Collar at the neck of the ulcer C: Projection from the lumen of the stomach D: All of the above Correct Ans:D Explanation The radiographic features that suggest a benign gastric ulcer include, • Projection from the lumen of the stomach • Smooth lucent line (Hampton line) or collar at the neck of the ulcer • Normal rugal folds that radiate to the edge of the ulcer collection • Complete and permanent healing of the ulcer on repeat radiographic or endoscopic examination of the stomach If at least two or more of these findings are present, a confident radiographic diagnosis of benign gastric ulcer is possible. Ref: Ott D.J. (2011). Chapter 10. Gastrointestinal Tract. In M.Y. Chen, T.L. Pope, D.J. Ott (Eds), Basic Radiology, 2e. Sample Previous Year Question on Radiological Signs based on previous Year Questions of NEET PG, USMLE,PLAB,FMGE (MCI Screening). Please visit www.medicoapps.org for more such Quizzes Which of the following statements best describes 'Background Radiation'?
  • 5. A: Radiation in the background of nuclear reactors B: Radiation in the background during radiological investigations C: Radiation present constantly from natural sources D: Radiation from nuclear fall out Correct Ans:C Explanation Background radiation refers to those coming from the environment of natural radioactivity at the earths surface and the those from direct cosmic radiation that arrives at the earths surface. Ref: Radiation Biophysics By Edward L. Alpen, Page 432; The Biological Basis of Nursing : Cancer By William T. Blows, Page 54, 56 Sample Previous Year Question on Radiological Signs based on previous Year Questions of NEET PG, USMLE,PLAB,FMGE (MCI Screening). Please visit www.medicoapps.org for more such Quizzes A 43 year old male during the preoperative evaluation of hernia repair is found to have a solitary pulmonary nodule. What is the important radiological feature to suggest its benign nature? A: Large lesion with calcification B: Irregular shape C: No calcification D: 1 cm lesion with uniform calcification Correct Ans:D Explanation The important radiological characteristics to suggest benign lesion in solitary pulmonary nodule: • No growth in follow up imaging in two years and calcification • Younger age group • Regular shape • Normal edge Ref: Lange radiology, Edition - 7, Page - 256.
  • 6. Sample Previous Year Question on Radiological Signs based on previous Year Questions of NEET PG, USMLE,PLAB,FMGE (MCI Screening). Please visit www.medicoapps.org for more such Quizzes Which of the following radiological sign will be present in a young lady with symptoms suggestive of pulmonary embolism? A: Hampton's hump B: Westermark sign C: Fleischner sign D: All of the above Correct Ans:D Explanation Pulmonary infarction may occur if the pulmonary venous pressure is elevated or the bronchial arterial supply to a region is deficient. The cone-shaped area of pulmonary infarction has been called a Hampton's hump. An area of radiolucency, corresponding to diminished pulmonary vascularity distal to a pulmonary embolism, is occasionally seen and is called the Westermark sign. There may also be an increase in the size of the pulmonary artery proximal to a large central pulmonary embolus (Fleischner sign). Ref: Chiles C., Gulla S.M. (2011). Chapter 4. Radiology of the Chest. In M.Y. Chen, T.L. Pope, D.J. Ott (Eds), Basic Radiology, 2e. Sample Previous Year Question on Radiological Signs based on previous Year Questions of NEET PG, USMLE,PLAB,FMGE (MCI Screening). Please visit www.medicoapps.org for more such Quizzes All of the following are radiological signs of scurvy, EXCEPT: A: Trimmerfeld zone B: Frenkel's line C: Pelican spur D: Soap bubble appearance Correct Ans:D Explanation Ascorbic acid is essential for the hydroxylation of lysine and proline in collagen, hence the impairment in wound healing. Collagen is an important part if bone also, so the defect in
  • 7. bones occurs. Soap bubble appearence in the xrays is usually seen in osteoblastoma and in gaint cell tumour. Ref: Terry Yochum, Lindsay Rowe (2005), Chapter 14, “Nutritional, Metabolic and Endocrine disorders”, In the Book, “Essentials of Skeletal Radiology”, Volume 1, 3rd Edition, USA, Page 1517 ; Text Book of Radiology and Imaging By Sutton, 7th Edition, Page 1356 Sample Previous Year Question on Radiological Signs based on previous Year Questions of NEET PG, USMLE,PLAB,FMGE (MCI Screening). Please visit www.medicoapps.org for more such Quizzes Which of the following condition gives the characteristic radiological finding "bone within a bone" appearance? A: Osteogenesis imperfecta B: Osteopetrosi s C: Scurvy D: Rickets Correct Ans:B Explanation Radiologic examination of osteopetrosis shows increased bone density and transverse bands in the shafts, clubbing of ends, and vertical striations of long bones. Thickening about the cranial foramina is present, and heterotopic calcification of soft tissues is possible. Other characteristic findings include a miniature bone inset within each vertebral body (bone-within-a bone appearance) and increased density at the end plates (sandwich vertebrae). Ref: Clinical Imaging: An Atlas of Differential Diagnosis By Ronald L. Eisenberg, 2012, Page 1088. Sample Previous Year Question on Radiological Signs based on previous Year Questions of NEET PG, USMLE,PLAB,FMGE (MCI Screening). Please visit www.medicoapps.org for more such Quizzes 20 years old female with complaints of nausea, vomiting, dizziness and pain in the legs. Her physical examination and all lab investigations and radiological investigations are normal. What would be the most probable diagnosis?
  • 8. A: Generalized anxiety disorder B: Conversion disorder C: Somatoform pain disorder D: Somatisation disorder Correct Ans:D Explanation Here the patient presents with multiple, medically unexplained symptoms (MUS). This is an example of Somatisation Disorder. In somatoform pain disorder the major symptom will be unexplained chronic pain, which is usually isolated. Patients have a long history of complaints of severe pain out of proportion to any biomedical findings that are present. Exacerbations and remission of complaints correlate with psychogenic factors The term somatization refers to the experience and reporting of physical symptoms that cause distress but lack a corresponding level of tissue damage or pathology and are linked to psychosocial stress. In contrast to this broad and inclusive view of the process, psychiatrists have developed strict diagnostic criteria that define several distinct disorders, which are collectively referred to as the somatoform disorders. As such, clinicians should be careful to distinguish between somatization, as defined above, and somatization disorder, which is one type of somatoform disorder. In general, these conditions are chronic and reflect an enduring way for the affected individuals to cope with psychosocial stressors. Ref: Shim J., Eisendrath S.J. (2008). Chapter 25. Somatization. In M.D. Feldman, J.F. Christensen (Eds), Behavioral Medicine: A Guide for Clinical Practice, 3e. Sample Previous Year Question on Radiological Signs based on previous Year Questions of NEET PG, USMLE,PLAB,FMGE (MCI Screening). Please visit www.medicoapps.org for more such Quizzes Radiological examination shows evidence of bone infarct in a child. She may have the following condition: A: Iron deficiency anaemia B: Thalassemi a C: Sickle cell disease D: Hereditary spherocytosis Correct Ans:C Explanation Sickle cell disease is frequently associated with bone infarction resulting from vasoocclusion secondary to the sickling of red cells. Bone infarction also occurs in hemoglobin sickle cell disease and sickle cell thalassemia. The bone pain in sickle cell crisis is due to bone and bone marrow infarction.
  • 9. In children, infarction of the epiphyseal growth plate interferes with normal growth of the affected extremity. Radiographically, infarction of the bone cortex results in periosteal elevation and irregular thickening of the bone cortex. Infarction in the bone marrow leads to lysis, fibrosis, and new bone formation. Ref: Langford C.A. (2012). Chapter 336. Arthritis Associated with Systemic Disease, and Other Arthritides. In D.L. Longo, A.S. Fauci, D.L. Kasper, S.L. Hauser, J.L. Jameson, J. Loscalzo (Eds), Harrison's Principles of Internal Medicine, 18e. Sample Previous Year Question on Radiological Signs based on previous Year Questions of NEET PG, USMLE,PLAB,FMGE (MCI Screening). Please visit www.medicoapps.org for more such Quizzes A 65-year-old man develops oliguria and peripheral edema over a period of weeks. Urinalysis reveals hematuria and proteinuria; examination of the urinary sediment reveals red cell casts. Radiological and ultrasound studies fail to demonstrate an obstructive lesion. Renal biopsy shows many glomerular crescents. This presentation is most suggestive of which of the following conditions? A: Anti-glomerular basement membrane disease B: Diabetic nephropathy C: Hypertensive nephropathy D: Lupus nephritis Correct Ans:A Explanation The two principal causes of rapidly progressive glomerulonephritis are anti-glomerular basement membrane (including both Goodpasture's syndrome and isolated anti-glomerular basement disease) and primary systemic vasculitis (including Wegener's granulomatosis, microscopic polyarteritis, idiopathic rapidly progressive glomerulonephritis, Churg-Strauss syndrome, polyarteritis nodosa, giant-cell arteritis, and Takayasu's arteritis). A very large variety of other systemic and primary glomerular disease may occasionally cause rapidly progressive glomerulonephritis, but this is usually not the typical presentation for these diseases. Diabetic nephropathy typically begins with microalbuminuria and hypertension and progresses over a 10 to 20 year period to renal failure. Hypertensive nephropathy due to essential hypertension typically presents with slowly rising BUN and creatinine; hypertensive nephropathy due to malignant hypertension presents with more rapidly rising BUN and creatinine. Lupus nephritis can have many presentations, but the most typical is proteinuria, which may be severe enough to cause nephrotic syndrome. Also, 90% of cases of systemic lupus
  • 10. erythematosus occur in women, usually of childbearing age. Ref: Lewis J.B., Neilson E.G. (2012). Chapter 283. Glomerular Diseases. In D.L. Longo, A.S. Fauci, D.L. Kasper, S.L. Hauser, J.L. Jameson, J. Loscalzo (Eds), Harrison's Principles of Internal Medicine, 18e Sample Previous Year Question on Radiological Signs based on previous Year Questions of NEET PG, USMLE,PLAB,FMGE (MCI Screening). Please visit www.medicoapps.org for more such Quizzes Which of the following condition is associated with radiological ‘Spalding sign'? A: Mummification B: Maceratio n C: Hanging D: Drowning Correct Ans:B Explanation ‘Spalding sign' is a sign of maceration. Loss of alignment and overriding of the bones of the cranial vault occur due to shrinkage of the cerebrum can be seen in X-ray after death of the foetus. The sign will develop earlier with a vertex presentation than with a breech. It may be detected within a few days of death of the foetus. Signs of dead birth other than maceration are rigor mortis at delivery and mummification. Ref: The Essentials of Forensic Medicine and Toxicology by KS Narayan Reddy, 27th edition, Page 382-383. Sample Previous Year Question on Radiological Signs based on previous Year Questions of NEET PG, USMLE,PLAB,FMGE (MCI Screening). Please visit www.medicoapps.org for more such Quizzes What is the ICRP (International Commission on Radiological Protection) recommended genetic dose of radiation exposure for general population? A: 5 rems over a period of 30 years B: 30 rems over a period of 30 years C: 5 rems over a period of 5 years D: 30 rems over a period of 5 years
  • 11. Correct Ans:A Explanation ICRP recommends that genetic dose to population should not exceed 5 rems over a period of 30 years. Ref: Park’s textbook of Preventive and Social Medicine, 21st edition, page-687 Sample Previous Year Question on Radiological Signs based on previous Year Questions of NEET PG, USMLE,PLAB,FMGE (MCI Screening). Please visit www.medicoapps.org for more such Quizzes One of the following disease will show urinary bladder calcification radiologically which resembles fetal head in pelvis: A: Tuberculosis B: Schistosomiasis C: Chronic cystitis D: Malignancy Correct Ans:B Explanation A plain film of the abdomen may show areas of grayness in the flank (enlarged hydronephrotic kidney) or in the bladder area (large tumor). Opacifications (stones) may be noted in the kidney, ureter, or bladder. Linear calcification may be seen in the ureteral and bladder walls. Punctate calcification of the ureter (ureteritis calcinosa) and a honeycombed calcification of the seminal vesicle may be obvious. The classic presentation of a calcified bladder, which looks like a fetal head in the pelvis, is pathognomonic of chronic urinary schistosomiasis. Ref: Tanagho E.A., Kane C.J. (2008). Chapter 14. Specific Infections of the Genitourinary Tract. In E.A. Tanagho, J.W. McAninch (Eds), Smith's General Urology, 17e. Sample Previous Year Question on Radiological Signs based on previous Year Questions of NEET PG, USMLE,PLAB,FMGE (MCI Screening). Please visit www.medicoapps.org for more such Quizzes All of the following are correct statements about radiological evaluation of a patient with Cushing's syndrome, EXCEPT: A: MRI of the sella tursica will always identify a pituitary cause for Cushing's syndrome
  • 12. B: Petrosal sinus sampling is the best way to distinguish a pituitary tumor from an ectopic ACTH producing tumor C: MRI of the adrenals may distinguish adrenal adenoma from carcinoma D: Adrenal CT scan distinguished adrenal cortical hyperplasia from an adrenal tumor Correct Ans:A Explanation Most ACTH-secreting pituitary tumors are <5 mm in diameter, and about half are undetectable by sensitive MRI. So MRI is not a useful investigation in identifying pituitary cause of Cushing's syndrome. Ref: Harrison’s Internal Medicine, 18th Edition, Chapter 339; CURRENT Diagnosis and Treatment: Surgery, 13th Edition, Chapter 33 Sample Previous Year Question on Radiological Signs based on previous Year Questions of NEET PG, USMLE,PLAB,FMGE (MCI Screening). Please visit www.medicoapps.org for more such Quizzes Radiological appearance of ‘corkscrew oesophagus’ is a characteristic finding seen in the condition: A: Achalasia cardia B: Diffuse esophageal spasm C: Carcinoma esophagus D: Reflux oesophagitis Correct Ans:B Explanation The lower esophageal sphincter in patients with diffuse esophageal spasm (DES) usually shows a normal resting pressure and relaxation on swallowing. A hypertensive sphincter with poor relaxation may also be present. In patients with advanced disease, the radiographic appearance of tertiary contractions appears helical, and has been termed corkscrew esophagus or pseudodiverticulosis. Patients with segmental or diffuse esophageal spasm can compartmentalize the esophagus and develop an epiphrenic or mid esophageal diverticulum between two areas of high pressure occurring simultaneously. Ref: Jobe B.A., Hunter J.G., Peters J.H. (2010). Chapter 25. Esophagus and Diaphragmatic Hernia. In F.C. Brunicardi, D.K. Andersen, T.R. Billiar, D.L. Dunn, J.G. Hunter, J.B. Matthews, R.E. Pollock (Eds), Schwartz's Principles of Surgery, 9e. Sample Previous Year Question on Radiological Signs based on previous Year Questions of NEET PG, USMLE,PLAB,FMGE (MCI Screening). Please visit www.medicoapps.org for more such Quizzes
  • 13. A 50 year old female is admitted with abdominal pain and anuria. Radiological studies revealed bilateral impacted ureteric stones with hydronephrosis. Urine analysis showed RBCs with pus cells in urine. Serum creatinine level was 16 mg/dl and urea level was 200 mmol/1. Which of the following should be the immediate treatment? A: Hemodialysis B: 'J' stent drainage C: Lithotripsy D: Ureteroscopic removal of stones Correct Ans:B Explanation This is the best modality of treatment to immediately revert anuria in patients with bilateral obstructing renal stone. Stenting can be done in other scenarios such as pyelonephritis due to obstructed stone, severe renal colic, long term obstruction. Ref: Urinary Stone Disease : The Practical Guide to Medical and Surgical Management By Marshall L. Stoller, Maxwell V. Meng, 2007, Page 475 Sample Previous Year Question on Radiological Signs based on previous Year Questions of NEET PG, USMLE,PLAB,FMGE (MCI Screening). Please visit www.medicoapps.org for more such Quizzes A 50 year old female is admitted with abdominal pain and anuria. Radiological studies revealed bilateral impacted ureteric stones with hydronephrosis. Urine analysis showed RBCs with pus cells in urine. Serum creatinine level was 16 mg/dl and urea level was 200 mmol/1. Which of the following should be the immediate treatment? A: Hemodialysis B: 'J' stent drainage C: Lithotripsy D: Ureteroscopic removal of stones Correct Ans:B Explanation 'J' stent drainage is the best modality of treatment to immediately revert anuria in patients with bilateral obstruction of renal stone. Stenting can be done in other scenarios such as pyelonephritis due to obstructed stone, severe renal colic, long term obstruction.
  • 14. Sample Previous Year Question on Radiological Signs based on previous Year Questions of NEET PG, USMLE,PLAB,FMGE (MCI Screening). Please visit www.medicoapps.org for more such Quizzes All of the following are correct statements about radiological evaluation of a patient with Cushing's syndrome, EXCEPT: A: MRI of the sella tursica will always identify a pituitary cause for Cushing's syndrome B: Petrosal sinus sampling is the best way to distinguish a pituitary tumor from an ectopic ACTH producing tumor C: MRI of the adrenals may distinguish adrenal adenoma from carcinoma D: Adrenal CT scan distinguished adrenal cortical hyperplasia from an adrenal tumor Correct Ans:A Explanation Most ACTH-secreting pituitary tumors are <5 mm in diameter, and about half are undetectable by sensitive MRI. So MRI is not a useful investigation in identifying pituitary cause of Cushing's syndrome. Ref: Harrison’s Internal Medicine, 18th Edition, Chapter 339; CURRENT Diagnosis and Treatment: Surgery, 13th Edition, Chapter 33 Sample Previous Year Question on Radiological Signs based on previous Year Questions of NEET PG, USMLE,PLAB,FMGE (MCI Screening). Please visit www.medicoapps.org for more such Quizzes Which of the following is the best radiological investigation in a case of suspected splenic rupture? A: CT B: USG C: MRI D: Peritoneal rupture Correct Ans:A Explanation CT can clearly show the extend of splenic injury. It can also distinguish the viable portion of the spleen.
  • 15. Sample Previous Year Question on Radiological Signs based on previous Year Questions of NEET PG, USMLE,PLAB,FMGE (MCI Screening). Please visit www.medicoapps.org for more such Quizzes A 45 year old female presented with a breast lump. You are suspecting carcinoma in this patient and want to do a triple assessment. Assessment: Triple assessment is a combination of clinical assessment, radiological imaging and a tissue sample taken for either cytological or histological analysis. Reason: The positive predictive value of this combination will exceed 99.9%. A: Both Assertion and Reason are true, and Reason is the correct explanation for Assertion B: Both Assertion and Reason are true, and Reason is not the correct explanation for Assertion C: Assertion is true, but Reason is false D: Assertion is false, but Reason is true Correct Ans:A Explanation In any patient who presents with a breast lump or other symptoms suspicious of carcinoma, the diagnosis should be made by triple assessment. Ref: Bailey & Love’s Short Practice of Surgery, 24th Edition, Page 826. Sample Previous Year Question on Radiological Signs based on previous Year Questions of NEET PG, USMLE,PLAB,FMGE (MCI Screening). Please visit www.medicoapps.org for more such Quizzes A 2 month old infant is presented with failure to thrive, recurrent emesis, hepatosplenomegaly, and adrenal insufficiency. Adrenal calcification is noted radiologically. What is the most likely diagnosis? A: Adrenal hemorrhage B: Wolman's disease C: Pheochromocytoma D: Addison's disease Correct Ans:B
  • 16. Explanation This child is showing features of Wolman disease. Wolman disease appear in the first few weeks of life and presents with persistent vomiting and diarrhea, hepatosplenomegaly, xanthomatosis and adrenal calcification. Patients have a complete absence of enzyme A of lysosome acid lipase. Investigations: shows liver enzyme abnormalities, decreased adrenal responsiveness to ACTH stimulation, normal or decreased plasma lipids. Cholesterol ester and triglyceride deposition occur in the lysosomes of liver parenchymal and Kuppfer cells, and in macrophages of adrenal gland, lymph node, intestinal mucosa etc. Sample Previous Year Question on Radiological Signs based on previous Year Questions of NEET PG, USMLE,PLAB,FMGE (MCI Screening). Please visit www.medicoapps.org for more such Quizzes Shenton’s line is a radiological line used to determine the pathology of: A: Hip B: Ankle C: Elbow D: Shoulder Correct Ans:A Explanation Shenton’s line is an imaginary semi-circular line joining medial cortex of the femoral neck to lower border of the superior pubic ramus. Shenton’s line is broken in posterior dislocation of the hip. Ref: Essential Orthopedics by Maheshwari, 3rd edition, Page 111. Sample Previous Year Question on Radiological Signs based on previous Year Questions of NEET PG, USMLE,PLAB,FMGE (MCI Screening). Please visit www.medicoapps.org for more such Quizzes A sixty five year old lady presents with a long standing history of pain and swelling in her right knee. Pain is significantly interfering with her activities of daily living. Radiological evaluation shows grade III changes of osteoarthritis. Which of the following is the recommended line of management?
  • 17. A: Conservative management B: Arthroscopic lavage/washout C: Partial knee replacement D: Total knee replacement Correct Ans:D Explanation This patients history strongly supports for a total knee replacement. As she is sixty five years old and radiological reports showing evidence of grade III changes of osteoarthritis in this patient, the best recommendation would be a Total knee arthroplasty. Patients with severe symptomatic Osteoarthritis have failed to respond to medical therapy and have progressive limitations in their daily activities should be referred for surgical options such as arthroplasty or joint reconstruction like osteotomy or arthrodesis. Ref: Osteoarthritis: Diagnosis and medical/surgical management, by Roland W.Moskowitz, Page 402, 403. Sample Previous Year Question on Radiological Signs based on previous Year Questions of NEET PG, USMLE,PLAB,FMGE (MCI Screening). Please visit www.medicoapps.org for more such Quizzes A child with a history of pain over lower extremities since few moths shows radiological evidence of osteosarcoma. Most common site of osteosarcoma is: A: Upper end of femur B: Lower end of femur C: Lower end of fibula D: Lower end of tibia Correct Ans:B Explanation The classic form of osteosarcoma is typically seen in patients in their second or third decade, with a peak in the adolescent growth spurt. It occurs more frequently in males than in females and is found in the metaphyseal areas of long bones, with 50% of lesions seen about the knee joint. The distal femur is the most common site, followed by the proximal tibia and then the proximal humerus.
  • 18. Ref: Randall R.L., Hoang B.H. (2006). Chapter 6. Musculoskeletal Oncology. In H.B. Skinner (Ed), CURRENT Diagnosis & Treatment in Orthopedics, 4e. Sample Previous Year Question on Radiological Signs based on previous Year Questions of NEET PG, USMLE,PLAB,FMGE (MCI Screening). Please visit www.medicoapps.org for more such Quizzes Radiological features of left ventricular heart failure are all, EXCEPT: A: Kerley B lines B: Cardiomegal y C: Oligemic lung fields D: Increased flow in upper lobe veins Correct Ans:C Explanation Classic signs of acute heart failure that can be seen on a chest x-ray of left ventricular failure are: • Cardiomegaly • Upper lobe blood diversion • "Bat's wing" alveolar edema • Pleural effusions Echocardiography is the investigation of choice and can identify and quantify LVH and dysfunction (both systolic and diastolic) as well as examine causes of heart failure, such as valve abnormalities. Oligemic lung fields seen in Pulmonary atresia, stenosis; Ebstein’s anomaly. Also Know: Chest X ray Findings • Boot shaped heart - TOF • “3”like appearance - Coarctation of aorta. • Figure of ‘8’- TAPVC(Total anomalous pulmonary venous connection) • Bat wings appearance -Pulmonary edema. Ref: O'Rourke R.A., Gilkeson R.C. (2011). Chapter 17. Cardiac Radiography. In V. Fuster, R.A. Walsh, R.A. Harrington (Eds), Hurst's The Heart, 13e.
  • 19. Sample Previous Year Question on Radiological Signs based on previous Year Questions of NEET PG, USMLE,PLAB,FMGE (MCI Screening). Please visit www.medicoapps.org for more such Quizzes